NBME 4 discussion

This forum made possible through the generous support of SDN members, donors, and sponsors. Thank you.

britesky89

Full Member
7+ Year Member
Joined
Apr 6, 2015
Messages
65
Reaction score
25
Hi guys, I totally bombed NBME 4. I only found threads that either discuss the scoring or that have an answer without explanations. I didn't find a thread that discusses the answers. So here goes: I have a lot of questions and I know its a lot to ask and I would be grateful for any help. I feel so depressed. I failed with a 206 (350) even after doing uworld twice with 73% the second time. Please help me!

1. 46 yo F, difficulty sleeping for 2 months. She worries about work causing her to wake up at 3 AM despite being tired. Has a hx of unstable angina requiring stent placement in the coronary artery twice. Has had chest pain that occurs daily since onset 1 mo ago. No organic cause has been found for the chest pain. She admits to being stress at work where she is an attorney. She has more responsibities now and doesn't enjoy work like she used to and reports difficulty paying attention. She is also a gourmet cook, but doesn't enjoy cooking like she used to because it doesn't taste good to her. Physical exam is normal. On mental status exam she is oriented to time, place, and person and appears anxious with a reactive affect. She admits she feels despondent a lot of the time, particularly when she thinks about her heart disease. Diagnosis?

a. Adjustment disorder (INCORRECT)
b. Dysthymic disorder
c. Generalized Anxiety disorder
d. Major depressive disorder
e. Primary Insomnia

2. 25 yo F, prenatal exam at 12 weeks gestation. She has been taking haloperidol for schizoaffective disorder until 2 mo ago. She believes the baby is evil and hears voices telling her to get an abortion. Uterus is consistent with a 12 week gestation on exam. Mental status exam shows an agitated woman. You decide to admit the patient. What additional intervention is the most appropriate next step in management?

a. Defer the decision about abortion and observe the patient. (INCORRECT)
b.Defer the decision about abortion and start valproic acid
c. Defer the decision about abortion and resume haloperidol
d. Arrange for abortion and begin valproic acid
e. Arrange for abortion and resume haloperidol
f. Arrange for the abortion and haloperidol after the abortion

3. 19 yo F, primigravid at 40 weeks gestation and uncomplicated pregnancy so far. Fetal heart monitor shows a baseline of 140/min with good variability. In 3o min her rate increases to 160/min for 25 to 30 sec. Next step in management?

a. Reassurance
b. Biophysical profile
c. Oxytocin challenge test
d. Induction of labor (INCORRECT)
e. C-section

4. 52 yo M, 30 lb weight loss over in 6 mo, oily, floating stools since 2 mo, diagnosed with acute pancreatitis 2 yrs ago with 1-3 episodes of severe abdominal pain since then. Patient takes oxycodone. Fx: mother has DM2, father died of alcoholic cirrhosis. Patient is an alcoholic but has been sober for past 2 years. He has a smoking history with 1 pack/daily for 30 yrs. BMI is 30. Vitals are normal. Abdomen in scaphoid with diffuse mild tenderness. Liver edge is firm and is palpated 2 cm below coastal margin. There are a bunch of lab values. I'll just mention a few of them: MCV 83, leukocytes 10,300, Na+ 139, K+ 3.5, HCO3 19, Glucose 164, ALP 120, AST 23, ALT 29, Amylase 90, Ferritin 250, Lipase 43 (N: 14-280). Next step in managment?

a. Dietary supplementation with a multivitamin with iron (INCORRECT)
b. Gluten free diet
c. Insulin therapy
d. Pancreatic enzyme replacement the
e. Parenteral nutrition

5. 16 yo F, yr hx of heavy bleeding with menses. Has never had spontaneous bleeding but has hx of excessive bleeding following dental procedure. Fx; father has hx of post op bleeding and nose bleeds; no bleeding disorder in mother, sister, and brother. Exam shows pallor. Labs: Hb 8, Hct 25%,, leukocyte 7000, Reticulocyte 2%, platelets 200,000, Bleeding time: 12 min, PT 13 sec with INR=1, PTT 60 sec. No abnormalities on Pelvic U/S. Mechanism of patient's bleeding?

a. Abnormal structure of Von Willebrand factor
b. Autoimmune platelet destruction
c. Bone marrow failure
d. capillary fragility
e. delay in megakaryocyte maturation
f. inadequete platelet production of prostacylin
g. inadequete production of factor VIII (INCORRECT)

6. 10 yo M with 7 day rash on arms and legs with BP 150/90. Rash is yellow, crusted, and excoriated over upper and lower extremities. Urine has 3+ protein, 30-50 RBC, 5-10 WBC. Diagnosis?

a. Acute Glomerulonephritis
b. Henoch Shonlein purpura (INCORRECT)
c. Lupus nephritis
d. Nephrolithiasis
e. Renal Neoplasm
f. UTI

7. 10 mo infant, 1 hr of labored breathing. For a duration of 18 hours has had fever, cough, coryzia. O2 saturation of 92% with bilateral crackles and wheezes. Vitals: HR 120/min, RR 54/min, BP 82/60, temp 102.2 F. Most likely pathophysiology?

a. Allergen induced bronchospasm (INCORRECT)
b. Barotrauma related alveolar disease
c. cardiac induced pulm edema
d. chemical irritant pneumonitis
e. Community acquired viral disease
f. contiguously spread bacterial infection
g. osmotically generated fluid shift
h. toxin mediated capillary leak

8. 82 yo F, hx of progressive urinary incontinence. Has strong urge to urinate at least 1x daily but is not able to reach bathroom on time, at which time she passes a large volume of urine. Other hx: arthritis of hips and knees that limits her mobility, cerebral infarction 2 yrs ago without residual weakness. Takes aspirin & lovastatin. Pelvic exam is normal. Cause of patient's incontinence?

a. Detrusor hyperactivity
b. Intrinsic weakness of urinary sphinter (INCORRECT)
c. outflow obstruction
d. poor pelvic support
e. UMN disease

9. 37 yo F, 3 mo of pain and masses in both breasts. Similar episodes that were not as severe have been occurring since 12 yrs. Use of OCPs for 16 yrs. Masses vary with menstrual cycles. Multinodular masses on exam. No adenopathy in axilla. Diagnosis?

a. benign cyst
b. breast abscess
c. breast carcinoma
d. breast engorgement
e. Ductal papilloma
f. fibroadenoma (INCORRECT)
g. Fibrocystic changes in breast
h. Mastitis
i. OCP induced breast changes

10. 32 yo G2P1 at 40 weeks, with confusion for 45 min. Has received all prenatal care from alternative provider, and has been in labor since past 3 days. Caregiver gave patient a natural product to chew this morning. resulting in strong, regular contractions. 6 hrs following this she lost conciousness suddenly. She is obtunded on arrival; vitals: 60 mmhg systolic, HR 140/min. She has a distended abdomen with rigidity. RUQ shows an irregular mobile mass that is 25 cm. Cervix is 50% effaced and 3 cm dilated with no fetal parts palpable. Cause of findings?

a. coagulopathy
b. endomyometritis
c. methamphetamine use (INCORRECT)
d. uterine atony
e. uterine rupture

11. 3 mo infant, 10 days of tacypnea and tachycardia with poor feeding. CXR: increased pulm markings & cardiomegaly, ECHO: VSD. Auscultation: 3/6 holosystolic murmur & 2/6 apical mid diastolic murmur. cause of findings?

a. excessive pulmonary blood flow
b. mitral v. obstruction
c. decreased LV contractility
d. decreased RV preload
e. RV pressure overload (INCORRECT)

12. 24 yo M, MVA with head trauma arrives comatose. He is intubated and mechanically ventilated. Vitals: HR 52/min, BP 160/94, temp 96 F. Roving eye movements. Corneal & pupillary reflexes normal. Spontaneous extention of legs and flexion of arms. DTRs are 3+ bilaterally. He is given 80 mL of 0.45% saline & urine output of 900 mL. Babinski +ve bilaterally. Labs: Na+ 147, Glucose 124, osmolality 294, urine specific gravity 1.001. CT scan: subarachnoid hemorrage & contusions. Cause of increased urine output?

a. diabetes insipidus
b. hypernatremia
c. SIADH
d. traumatic nephropathy (INCORRECT)
e. DM2

Members don't see this ad.
 
1. 46 yo F, difficulty sleeping for 2 months. She worries about work causing her to wake up at 3 AM despite being tired. Has a hx of unstable angina requiring stent placement in the coronary artery twice. Has had chest pain that occurs daily since onset 1 mo ago. No organic cause has been found for the chest pain. She admits to being stress at work where she is an attorney. She has more responsibities now and doesn't enjoy work like she used to and reports difficulty paying attention. She is also a gourmet cook, but doesn't enjoy cooking like she used to because it doesn't taste good to her. Physical exam is normal. On mental status exam she is oriented to time, place, and person and appears anxious with a reactive affect. She admits she feels despondent a lot of the time, particularly when she thinks about her heart disease. Diagnosis?

a. Adjustment disorder (INCORRECT) -There is no significant life stressor in the question.
b. Dysthymic disorder
c. Generalized Anxiety disorder
d. Major depressive disorder (SIGECAPS, Sad (despondent), low energy, poor appetite, anhedonia, 4/9 for 2+weeks)
e. Primary Insomnia

2. 25 yo F, prenatal exam at 12 weeks gestation. She has been taking haloperidol for schizoaffective disorder until 2 mo ago. She believes the baby is evil and hears voices telling her to get an abortion. Uterus is consistent with a 12 week gestation on exam. Mental status exam shows an agitated woman. You decide to admit the patient. What additional intervention is the most appropriate next step in management?

a. Defer the decision about abortion and observe the patient. (INCORRECT)
b.Defer the decision about abortion and start valproic acid
c. Defer the decision about abortion and resume haloperidol (She lacks decision making capability, Valproic Acid is a teratogen)
d. Arrange for abortion and begin valproic acid
e. Arrange for abortion and resume haloperidol
f. Arrange for the abortion and haloperidol after the abortion

3. 19 yo F, primigravid at 40 weeks gestation and uncomplicated pregnancy so far. Fetal heart monitor shows a baseline of 140/min with good variability. In 3o min her rate increases to 160/min for 25 to 30 sec. Next step in management?

a. Reassurance (research this on in another forum)
b. Biophysical profile
c. Oxytocin challenge test
d. Induction of labor (INCORRECT) (There is no risk for mother/fetus at the moment, and 42 weeks is the max for term)
e. C-section

4. 52 yo M, 30 lb weight loss over in 6 mo, oily, floating stools since 2 mo, diagnosed with acute pancreatitis 2 yrs ago with 1-3 episodes of severe abdominal pain since then. Patient takes oxycodone. Fx: mother has DM2, father died of alcoholic cirrhosis. Patient is an alcoholic but has been sober for past 2 years. He has a smoking history with 1 pack/daily for 30 yrs. BMI is 30. Vitals are normal. Abdomen in scaphoid with diffuse mild tenderness. Liver edge is firm and is palpated 2 cm below coastal margin. There are a bunch of lab values. I'll just mention a few of them: MCV 83, leukocytes 10,300, Na+ 139, K+ 3.5, HCO3 19, Glucose 164, ALP 120, AST 23, ALT 29, Amylase 90, Ferritin 250, Lipase 43 (N: 14-280). Next step in managment?

a. Dietary supplementation with a multivitamin with iron (INCORRECT)
b. Gluten free diet
c. Insulin therapy
d. Pancreatic enzyme replacement the (steatorrhea due to pancreatic enzyme insufficiency secondary to Chronic pancreatitis = acute pancreatitis 2yrs ago + 1-3 episode of severe abdominal pain since then)
e. Parenteral nutrition

5. 16 yo F, yr hx of heavy bleeding with menses. Has never had spontaneous bleeding but has hx of excessive bleeding following dental procedure. Fx; father has hx of post op bleeding and nose bleeds; no bleeding disorder in mother, sister, and brother. Exam shows pallor. Labs: Hb 8, Hct 25%,, leukocyte 7000, Reticulocyte 2%, platelets 200,000, Bleeding time: 12 min, PT 13 sec with INR=1, PTT 60 sec. No abnormalities on Pelvic U/S. Mechanism of patient's bleeding?

a. Abnormal structure of Von Willebrand factor (Increase BT, Increase PTT; vWF carries VIII, Most Common Inherited bleeding disorder via Autosomal Dominant fashion)
b. Autoimmune platelet destruction
c. Bone marrow failure
d. capillary fragility
e. delay in megakaryocyte maturation
f. inadequete platelet production of prostacylin
g. inadequete production of factor VIII (INCORRECT) (Hemophilias have Normal BT)

6. 10 yo M with 7 day rash on arms and legs with BP 150/90. Rash is yellow, crusted, and excoriated over upper and lower extremities. Urine has 3+ protein, 30-50 RBC, 5-10 WBC. Diagnosis?

a. Acute Glomerulonephritis (Hypertension, Impetigo, Proteinuria, RBC Casts)
b. Henoch Shonlein purpura (INCORRECT) (lower extremity purpuric rash)
c. Lupus nephritis
d. Nephrolithiasis
e. Renal Neoplasm
f. UTI

7. 10 mo infant, 1 hr of labored breathing. For a duration of 18 hours has had fever, cough, coryzia. O2 saturation of 92% with bilateral crackles and wheezes. Vitals: HR 120/min, RR 54/min, BP 82/60, temp 102.2 F. Most likely pathophysiology?

a. Allergen induced bronchospasm (INCORRECT)
b. Barotrauma related alveolar disease
c. cardiac induced pulm edema
d. chemical irritant pneumonitis
e. Community acquired viral disease (Fever, cough, coryza, Wheezes due to lower respiratory track infection = RSV Bronchiolitis)
f. contiguously spread bacterial infection
g. osmotically generated fluid shift
h. toxin mediated capillary leak

8. 82 yo F, hx of progressive urinary incontinence. Has strong urge to urinate at least 1x daily but is not able to reach bathroom on time, at which time she passes a large volume of urine. Other hx: arthritis of hips and knees that limits her mobility, cerebral infarction 2 yrs ago without residual weakness. Takes aspirin & lovastatin. Pelvic exam is normal. Cause of patient's incontinence?

a. Detrusor hyperactivity (urge incontinence)
b. Intrinsic weakness of urinary sphinter (INCORRECT)
c. outflow obstruction
d. poor pelvic support
e. UMN disease

9. 37 yo F, 3 mo of pain and masses in both breasts. Similar episodes that were not as severe have been occurring since 12 yrs. Use of OCPs for 16 yrs. Masses vary with menstrual cycles. Multinodular masses on exam. No adenopathy in axilla. Diagnosis?

a. benign cyst
b. breast abscess
c. breast carcinoma
d. breast engorgement
e. Ductal papilloma
f. fibroadenoma (INCORRECT)
g. Fibrocystic changes in breast (masses vary with menstrual cycle)
h. Mastitis
i. OCP induced breast changes

10. 32 yo G2P1 at 40 weeks, with confusion for 45 min. Has received all prenatal care from alternative provider, and has been in labor since past 3 days. Caregiver gave patient a natural product to chew this morning. resulting in strong, regular contractions. 6 hrs following this she lost conciousness suddenly. She is obtunded on arrival; vitals: 60 mmhg systolic, HR 140/min. She has a distended abdomen with rigidity. RUQ shows an irregular mobile mass that is 25 cm. Cervix is 50% effaced and 3 cm dilated with no fetal parts palpable. Cause of findings?

a. coagulopathy
b. endomyometritis
c. methamphetamine use (INCORRECT)
d. uterine atony
e. uterine rupture (hemorrhage ( SP of 60 mmhg = hemorrhage , irregular palpable mass = fetal feet, no fetal parts palpable in cervix = retraction of fetus)

11. 3 mo infant, 10 days of tacypnea and tachycardia with poor feeding. CXR: increased pulm markings & cardiomegaly, ECHO: VSD. Auscultation: 3/6 holosystolic murmur & 2/6 apical mid diastolic murmur. cause of findings?

a. excessive pulmonary blood flow (VSD with Eisenmeigers)
b. mitral v. obstruction
c. decreased LV contractility
d. decreased RV preload
e. RV pressure overload (INCORRECT)

12. 24 yo M, MVA with head trauma arrives comatose. He is intubated and mechanically ventilated. Vitals: HR 52/min, BP 160/94, temp 96 F. Roving eye movements. Corneal & pupillary reflexes normal. Spontaneous extention of legs and flexion of arms. DTRs are 3+ bilaterally. He is given 80 mL of 0.45% saline & urine output of 900 mL. Babinski +ve bilaterally. Labs: Na+ 147, Glucose 124, osmolality 294, urine specific gravity 1.001. CT scan: subarachnoid hemorrage & contusions. Cause of increased urine output?

a. diabetes insipidus ( He developed Central Diabetic Insipidus, No ADH, after head trauma)
b. hypernatremia
c. SIADH
d. traumatic nephropathy (INCORRECT)
e. DM2[/QUOTE]

Did this one a couple days ago, got them right on my test.
 
Last edited:
1. 46 yo F, difficulty sleeping for 2 months. She worries about work causing her to wake up at 3 AM despite being tired. Has a hx of unstable angina requiring stent placement in the coronary artery twice. Has had chest pain that occurs daily since onset 1 mo ago. No organic cause has been found for the chest pain. She admits to being stress at work where she is an attorney. She has more responsibities now and doesn't enjoy work like she used to and reports difficulty paying attention. She is also a gourmet cook, but doesn't enjoy cooking like she used to because it doesn't taste good to her. Physical exam is normal. On mental status exam she is oriented to time, place, and person and appears anxious with a reactive affect. She admits she feels despondent a lot of the time, particularly when she thinks about her heart disease. Diagnosis?

a. Adjustment disorder (INCORRECT) -There is no significant life stressor in the question.
b. Dysthymic disorder
c. Generalized Anxiety disorder
d. Major depressive disorder (SIGECAPS, Sad (despondent), low energy, poor appetite, anhedonia, 4/9 for 2+weeks)
e. Primary Insomnia

2. 25 yo F, prenatal exam at 12 weeks gestation. She has been taking haloperidol for schizoaffective disorder until 2 mo ago. She believes the baby is evil and hears voices telling her to get an abortion. Uterus is consistent with a 12 week gestation on exam. Mental status exam shows an agitated woman. You decide to admit the patient. What additional intervention is the most appropriate next step in management?

a. Defer the decision about abortion and observe the patient. (INCORRECT)
b.Defer the decision about abortion and start valproic acid
c. Defer the decision about abortion and resume haloperidol (She lacks decision making capability, Valproic Acid is a teratogen)
d. Arrange for abortion and begin valproic acid
e. Arrange for abortion and resume haloperidol
f. Arrange for the abortion and haloperidol after the abortion

3. 19 yo F, primigravid at 40 weeks gestation and uncomplicated pregnancy so far. Fetal heart monitor shows a baseline of 140/min with good variability. In 3o min her rate increases to 160/min for 25 to 30 sec. Next step in management?

a. Reassurance (research this on in another forum)
b. Biophysical profile
c. Oxytocin challenge test
d. Induction of labor (INCORRECT) (There is no risk for mother/fetus at the moment, and 42 weeks is the max for term)
e. C-section

4. 52 yo M, 30 lb weight loss over in 6 mo, oily, floating stools since 2 mo, diagnosed with acute pancreatitis 2 yrs ago with 1-3 episodes of severe abdominal pain since then. Patient takes oxycodone. Fx: mother has DM2, father died of alcoholic cirrhosis. Patient is an alcoholic but has been sober for past 2 years. He has a smoking history with 1 pack/daily for 30 yrs. BMI is 30. Vitals are normal. Abdomen in scaphoid with diffuse mild tenderness. Liver edge is firm and is palpated 2 cm below coastal margin. There are a bunch of lab values. I'll just mention a few of them: MCV 83, leukocytes 10,300, Na+ 139, K+ 3.5, HCO3 19, Glucose 164, ALP 120, AST 23, ALT 29, Amylase 90, Ferritin 250, Lipase 43 (N: 14-280). Next step in managment?

a. Dietary supplementation with a multivitamin with iron (INCORRECT)
b. Gluten free diet
c. Insulin therapy
d. Pancreatic enzyme replacement the (steatorrhea due to pancreatic enzyme insufficiency secondary to Chronic pancreatitis = acute pancreatitis 2yrs ago + 1-3 episode of severe abdominal pain since then)
e. Parenteral nutrition

5. 16 yo F, yr hx of heavy bleeding with menses. Has never had spontaneous bleeding but has hx of excessive bleeding following dental procedure. Fx; father has hx of post op bleeding and nose bleeds; no bleeding disorder in mother, sister, and brother. Exam shows pallor. Labs: Hb 8, Hct 25%,, leukocyte 7000, Reticulocyte 2%, platelets 200,000, Bleeding time: 12 min, PT 13 sec with INR=1, PTT 60 sec. No abnormalities on Pelvic U/S. Mechanism of patient's bleeding?

a. Abnormal structure of Von Willebrand factor (Increase BT, Increase PTT; vWF carries VII, Most Common Inherited bleeding disorder via Autosomal Dominant fashion)
b. Autoimmune platelet destruction
c. Bone marrow failure
d. capillary fragility
e. delay in megakaryocyte maturation
f. inadequete platelet production of prostacylin
g. inadequete production of factor VIII (INCORRECT) (Hemophilias have Normal BT)

6. 10 yo M with 7 day rash on arms and legs with BP 150/90. Rash is yellow, crusted, and excoriated over upper and lower extremities. Urine has 3+ protein, 30-50 RBC, 5-10 WBC. Diagnosis?

a. Acute Glomerulonephritis (Hypertension, Impetigo, Proteinuria, RBC Casts)
b. Henoch Shonlein purpura (INCORRECT) (lower extremity purpuric rash)
c. Lupus nephritis
d. Nephrolithiasis
e. Renal Neoplasm
f. UTI

7. 10 mo infant, 1 hr of labored breathing. For a duration of 18 hours has had fever, cough, coryzia. O2 saturation of 92% with bilateral crackles and wheezes. Vitals: HR 120/min, RR 54/min, BP 82/60, temp 102.2 F. Most likely pathophysiology?

a. Allergen induced bronchospasm (INCORRECT)
b. Barotrauma related alveolar disease
c. cardiac induced pulm edema
d. chemical irritant pneumonitis
e. Community acquired viral disease (Fever, cough, coryza, Wheezes due to lower respiratory track infection = RSV Bronchiolitis)
f. contiguously spread bacterial infection
g. osmotically generated fluid shift
h. toxin mediated capillary leak

8. 82 yo F, hx of progressive urinary incontinence. Has strong urge to urinate at least 1x daily but is not able to reach bathroom on time, at which time she passes a large volume of urine. Other hx: arthritis of hips and knees that limits her mobility, cerebral infarction 2 yrs ago without residual weakness. Takes aspirin & lovastatin. Pelvic exam is normal. Cause of patient's incontinence?

a. Detrusor hyperactivity (urge incontinence)
b. Intrinsic weakness of urinary sphinter (INCORRECT)
c. outflow obstruction
d. poor pelvic support
e. UMN disease

9. 37 yo F, 3 mo of pain and masses in both breasts. Similar episodes that were not as severe have been occurring since 12 yrs. Use of OCPs for 16 yrs. Masses vary with menstrual cycles. Multinodular masses on exam. No adenopathy in axilla. Diagnosis?

a. benign cyst
b. breast abscess
c. breast carcinoma
d. breast engorgement
e. Ductal papilloma
f. fibroadenoma (INCORRECT)
g. Fibrocystic changes in breast (masses vary with menstrual cycle)
h. Mastitis
i. OCP induced breast changes

10. 32 yo G2P1 at 40 weeks, with confusion for 45 min. Has received all prenatal care from alternative provider, and has been in labor since past 3 days. Caregiver gave patient a natural product to chew this morning. resulting in strong, regular contractions. 6 hrs following this she lost conciousness suddenly. She is obtunded on arrival; vitals: 60 mmhg systolic, HR 140/min. She has a distended abdomen with rigidity. RUQ shows an irregular mobile mass that is 25 cm. Cervix is 50% effaced and 3 cm dilated with no fetal parts palpable. Cause of findings?

a. coagulopathy
b. endomyometritis
c. methamphetamine use (INCORRECT)
d. uterine atony
e. uterine rupture (hemorrhage ( SP of 60 mmhg = hemorrhage , irregular palpable mass = fetal feet, no fetal parts palpable in cervix = retraction of fetus)

11. 3 mo infant, 10 days of tacypnea and tachycardia with poor feeding. CXR: increased pulm markings & cardiomegaly, ECHO: VSD. Auscultation: 3/6 holosystolic murmur & 2/6 apical mid diastolic murmur. cause of findings?

a. excessive pulmonary blood flow (VSD with Eisenmeigers)
b. mitral v. obstruction
c. decreased LV contractility
d. decreased RV preload
e. RV pressure overload (INCORRECT)

12. 24 yo M, MVA with head trauma arrives comatose. He is intubated and mechanically ventilated. Vitals: HR 52/min, BP 160/94, temp 96 F. Roving eye movements. Corneal & pupillary reflexes normal. Spontaneous extention of legs and flexion of arms. DTRs are 3+ bilaterally. He is given 80 mL of 0.45% saline & urine output of 900 mL. Babinski +ve bilaterally. Labs: Na+ 147, Glucose 124, osmolality 294, urine specific gravity 1.001. CT scan: subarachnoid hemorrage & contusions. Cause of increased urine output?

a. diabetes insipidus ( He developed Central Diabetic Insipidus, No ADH, after head trauma)
b. hypernatremia
c. SIADH
d. traumatic nephropathy (INCORRECT)
e. DM2

Did this one a couple days ago, got them right on my test.[/QUOTE]
I concur.
 
I have some more questions. I'm sorry I know its a lot. I couldn't find any other resources with answers, and I am really confused about these questions. I would be extremely grateful if I could get help with them. It would be great if you guys could help with what ever questions you know, even if its not all of them. Thanks in advance.




77 yo F, severe SOB after colectomy 3 days ago. Current meds: subcutaneous heparin & patient-controlled morphine. HR 140/min, resp 28/min, temp 98.6 F, BP 110/60, O2 sat: 89%. Breath sounds are reduced on right. Surgical incision as no discharge or erythema. ABGs: pH 7.38, PCO2 23, PO2 55. Atelectasis at both lung bases on CXR.
Nonspecific ST-T wave changes on EKG. Besides O2 theraphy, next step in mangment?

a. Spiral CT scan of chest
b. oral warfarin therapy
c. IV recombinant tPA
d. Pulm angiography (INCORRECT)
e. Placement of IVC filter

37 yo M who returned form a trip to Mexico 1 mo ago, with fever & bilateral flank pain. Takes no meds & has fever (102.2 F). Spleen is palpable 2 cm under coastal margin with deep inspiration. Hb 9, reticulocytes 5%, LDH 300. Next step in Diagnosis?

a. Aspiration of lesion
b. examination of duodenal aspirate for ova & parasites
c. examination of stool for ova & parasites (INCORRECT)
d. serum antibody titer for Entameba histolytica
e. serum antibody titer for Plasmodium species
f. serum antibody titer for strongyloides stercoralis
g. serum antibody titer for visceral larva migrans
h. skin test for trichinella spiralis
i. thick and thin blood smears for malaria

77 yo F 24 hr hx of watery diarrhea & nausea is admitted. So far during her stay she has vomited 4x and has not been able to drink or eat. Patient has hx of HTN, DM2, hyperlipidemia, chronic renal insufficiency. She is taking atorvastatin, lisinopril, HCTZ, & aspirin. She has dry mucous membranes. Abdomen is non-tender & soft. Vitals: BP 100/60, HR 110/min, temp 99.3 F, RR 22/min. Labs: Na+ 132, K+ 8.2, Cl- 99, HCO3 18, BUN 95, Creatinine 6.4, Glucose 199. Peaked T waves and QRS of 0.16 on EKG. Next step in managment?

a. administer bicarbonate
b. administer calcium gluconate
c. administer insulin and 50% dextrose in water
d. administer sodium polystyrene sulfonate (Kayexalate)
e. Schedule dialysis (INCORRECT - I thought she had hyperkalemia and renal failure so that was an indication for dialysis???)
,
23 yo F with 4 days hx of abd discomfort, loss of appetite, nausea. There is diffuse abd tenderness to deep palpation. RR 28/min. Labs: Na+ 132, Cl- 102, K+ 6, HCO3 10, Glucose 450. Cause?

a. addison's disease (INCORRECT)
b. diabetes insipidus
c. diabetic ketoacidosis
d. hemolysis
e. laxative abuse
f. lower GI tract fistula
g. renal tubular acidosis
h. SLE
i. tumor lysis synd
j. vomiting

25 yo F, 4 hr hx of SOB, marked malaise, weakness, & fatigue. 24 hrs ago she had a strenous physical workout follwoed by muscle soreness. She took ibuprofen for her pain 1 hr prior to onset of her symptoms. Has hx of migraines for which she's been metoprolol 2x daily for pro[hylaxis over the past month. Exam: mild periorbital edema & facial flushing. There are multiple wheezes through out lung fields. BP 100/70. Substance to prevent recurrence of similar episode?

a. acetaminophen (INCORRECT)
b. aspirin
c. meperidine
d. prednisone
e. propoxyphene

87 yo nursing home resident, with fever for 1 day. She has urinary incontinence for which a catheter was placed 2 weeks ago. Hx of Alzheimers dementia and cannot verbally communicate. Has moist and pink mucous membranes. Vitals: HR 86/min, BP 120/74, resp 14/min, temp 100 F. Urinanalysis: cloudy brown, pH 8.8, blood 2+,
Glucose -ve, protein 2+, RBC: too numerous to count, WBC 20-25, Nitrites 3+, Leukocyte esterase 3+, bacteria: many. Gram -ve bacilli on gram stain. Measure most likely to prevent patient's current condition?

a. use of incontinence briefs instead of catheter
b. changing catheter daily (INCORRECT)
c. cleaning urethral orifice with povidone iodide daily
d. flushing the catheter with antibiotic solution daily
e. oral antibiotic prophylaxis
f. oral oxybutynin therapy

57 yo F, hx of progressive cough productive of 2-3 tablespoons of yellow sputum daily for 2 yrs. It lasts all day but it worse in the morning. She is unable to walk more than one block before becoming short of breath. No hx of fever, hemoptysis, weight loss, or night sweats. Smoking hx of 2 packs daily for 40 yrs. BMI 32. BP 150/90, HR 96/min, RR 20/min. Has an O2 saturation of 90%. Breath sounds are decreased in all lung fields with rhonchi heard at bases and a prolonged expiratory phase. She has sinus tachycardia and P pulmonale on EKG. PFTs show FEV1 greater than 80% of predicted, & FEV1/FVC ratio of 70% of predicted and a diffusion lung capacity of carbon monoxide greater than 60 % of predicted. Diagnosis?

a. chronic bronchitis
b. cystic fibrosis
c. emphysema (INCORRECT - I thought only emphysema had an increased CO diffusion capacity?)
d kartegener's syndrome
e. post nasal drip syndrome

42 yo M, MVA, undergoes splenectomy and one hour later has severe SOB. Other injuries are left rib and pelvic fractures. Vitals: BP 80/60, HR 133/min, RR 35/min, temp 97.4 F. He has slight distention & absent bowel sounds on abd exam. Next step in management.

a. Xray of chest
b. transfusion type specific uncrossedmatched blood
c. IV heparin therapy
d. fiberoptic bronchoscopy
e. endotracheal intubation (INCORRECT)
f. placement of vena cava filter
g. needle thoracostomy


72 yo F 3 mo hx of bilateral knee pain with weight bearing. OTC naproxen at maximum dose over past 6 weeks has partially releived the pain. Has Hx of HTN, CAD, CHF, & osteoporosis. Also has hx of stable exertional angina & moderate SOB with exertion. She is taking atenolol, potassium supplements, furosemide. BP 115/70, HR 62/min. She has JVD and lungs are clear to auscultation. CVS exam shows no murmurs or gallops but she has pedal edema. She has crepitus of both knees, and bony prominences at proximal tibial bone on both sides. Labs show potassium of 5.1, BUN of 47 (baseline 20), creatinine of 2.2 (baseline of 1). Which medication is responsible for abnormal lab findings?

a. atenolol
b. furosemide
c. naproxen
d. nitrates
e. potassium supplements (INCORRECT)

67 yo alcoholic male comes for routine exam. Has a 15 yr hx of poorly controlled HTN for which he takes HCTZ but he is not compliant. His blood pressure at the current visit is 170/102 and was at this level for the last 3 office visits. Funduscope findings: AV nicking & tortuosity of the arteries. Patient is at greatest risk for?

a. aortic aneurysm (INCORRECT)
b. glaucoma
c. myocardial infarction
d. pulmonary HTN
e. subarachnoid hemorrage

67 yo African American F comes for routine exam. She reached menopause at age 44. She has hypothyroidism and is taking thyroid replacement therapy. Has smoking hx of 1 pack/daily for 45 yrs. BMI 21. Which factor decreases patient's risk for osteoporosis?

a. African American race
b .cigarette smoking
c. early menopause
d. thin body habitus
e. thyroid replacement therapy (INCORRECT)

37 yo F with known ITP has sudden onset of fever 7 days after undergoing splenectomy. She has had 2 days of mild shoulder pain & moderate abd pain. Hb has been between 9.5 and 10 since the splenectomy. Currently she is on hydrocodone and docusate. She has a fever of 102.4 F. Abd Exam; mild distention & diffuse tenderness without rebound, rigidity, or guarding, and absent bowel sounds. Labs: Hb 9.8, Leukocyte ct. 21,300, segmented neutrophils 50%, bands 17%, eosinophils 3%, lymphocytes 25%, monocytes 5%, platelet ct 105,000, amylase 124. Xray shows left pleurual effusion. Cause of findings?

a. aspiration
b. gastric perforation (INCORRECT)
c. pancreatitis
d. pneumonia
e. subphrenic abscess

82 yo M, lives in nursing home: SOB at rest, increasing malaise, generalized weakness, & depressed mood over past 6 weeks. He bruises easily and has bleeding & inflammation of the gums. He reports a minor skin abrasion 1 month ago that has not healed completely, and he appears chronically ill. Exam: numerous petechiae, eccymosis, & hypertrophic lichenified patches over upper extremities. Mild pedal edema is present. He is oriented to TPP and has no short or long term memory loss. Labs including CBC, urinanalyis, measurement of serum electrolyte concentrations, are all with in normal limits. Diagnosis?

a. Acquired factor VIII deficiency (INCORRECT)
b. hypersensitivity vasculitis
c. polymyalgia rheumatica
d. psoriasis
e. thrombocytopenic purpura
f. vit B12 deficiency
g. vit C deficiency

Asymptomatic 72 yo F for routine exam, takes no meds. DEXA findings at age 65 were normal. At her last visit 1 yr ago pap smear, mammogrpahy, sigmoidoscopy, were all normal. Her serum cholesterol was 190 and glucose was 80. She remains sexually active and does not smoke. She drinks 1-2 glasses of wine 3x weekly. BMI 23. BP 140/85. Physical and pelvic exams show no abnormalities. Next step in managment?

a. measurment of fasting glucose concentration
b. serum lipid studies (INCORRECT)
c. ECG
d. Influenza virus vaccination
e. sigmoidoscopy

72 yo lethargic appearing dementia patient who lives in a nursing home brought to ER 2 days after onset of fever, confusion, and generalized weakness. Symptoms began following irrigation of his urinary catheter, which wasn't draining well. He has hx of COPD and osteoarthritis. Vitals: Temp 97.2 F, HR 110, BP 90/50. Has supple neck on exam. Herbenden nodes are seen on the hands. Breath sounds are decreased throughout lungs with no dullness. Cranial nerves are intact and DTRs are normal. Labs: Hct 50%, leukocyte ct 15,400, segmented neutrophils 74%, bands 4%, lymphocytes 22%, Na+ 134, K+ 4.2, Cl- 86, HCO3 24, BUN 12, creatinine 1.1. Sinus tachycardiea and some flattening of ST-T segments. Most likely explanation for hypotension?

a. decreased cardiac contractility
b. decreased systemic vascular resistance
c. deficiency of mineralcorticoids
d. impaired vagal reflexes
e. impaired venous return (INCORRECT)

Follow up of a 6 mo boy who's had chronic constipation since age 1 week. Current treatment with rectal stimulation, glycerin suppositories, & 4 oz of prune juice produces one firm string like stool every 4 days. After being breast fed until 2 weeks of age he was switched to cow's milk-based formula. At the moment his diet also consists of cereals, fruits, pureed vegetables. Abd exam: distention with no tenderness. Rectal exam shows no palpable stool in ampulla. Next step in management?

a. switch to lactose-free formula
b. test of stool for botulism toxin (INCORRECT)
c. stool culture
d. measurement of serum TSH concentration
e. rectal manometry
f. upper GI series

52 yo F, routine exam. She was started on daily hormone replacment therapy (estradiol and medroxyprogesterone) one month ago to treat severe vasomotor symptoms. But she has been taking the medroxyprogesterone intermittently due to severe depression and mood changes while on it. For the first 6 mo of the regimen menses had stopped. However, since she changed her regimen she has vaginal spotting 2 -3x/ monthly. Next step in managment?

a. discontinue medroxyprogesterone therapy only
b. increase dosage of estradiol
c. add an antidepressant to the medication regimen (INCORRECT)
d. endometrial abalation
e. endometrial biopsy

3 yo girl, 1 day hx of fever and ear pain, has had clear nasal discharge and cough for 3 days. Over last 12 months she has had a hx of several ear infections and one episode of strep pharangitis. Her family has two cats and the father smokes in the home. She frequently swims in the family's swimming pool. BP 80/50, HR 110/min, RR 22/min, temp 101.3 F. She has clear nasal discharage, erythemea and bulging of right tympanic membrane and erythema of throat without exudate. Lungs are clear to auscultation. Most appropriate recommendation to prevent recurrence of this patient's condition?

a. avoidance of passive smoke exposure
b. removal of cats from the house
c. use of earplugs while swimming (INCORRECT)
d. daily otic antibiotics
e. prophylactic oral decongestants
f. tonsilectomy

67 yo F, 2 mo hx of intermittent burning sensation in her epigastrium. It presents after she walks one block or climbs one flight of stairs and is sometimes associated with lightheadedness, and profuse sweating. Epigastric discomfort is better with rest and is not relieved by antacids. Her meds include daily multivitamin and calcium supplements. Smoker for 40 yrs with 1 pack/daily. She has one drink of alcohol 1-2x/ weekly. BP 150/90, HR 98/min, resp 15/min, temp 98.6F. Results of labs, including cardiac enzymes, are within normal range. No abnormalities are noted on EKG. Most appropriate next step in diagnosis?

a. ambulatory EKG monitoring (INCORRECT)
b. abdominal U/S
c. Cardiac stress schintography
d. 24 hr monitoring of esophageal pH
e. esophagogastroduodenoscopy

62 yo M brought 4 hrs after the sudden onset of vomiting & severe abd pain. Appears diaphoretic and in acute distress. He has had mild,intermittent epigastric pain over past year that is relieved by antacids. No hx of serious illness. He is a smoker : 1 pack/daily for 35 yrs and has 1-2 drinks/ daily. BP 115/75, HR 100/min, RR 18/min, temp 101.1 F. Abd is rigid and tympanic to percussion over the liver, with severe diffuse tenderness. No bowel sounds are present. Next step in diagnosis?

a. Xrays of chest and abdomen
b. barium enema
c. water soluble contrast swallow
d. CT scan of abd (INCORRECT)
e. colonoscopy
f. flexible esophagogastroduodenoscopy

32 yo F, 2 day hx of fever & an area of swelling & redness on her right forearm. She noticed red streaks extending from her forearm to her elbow 6 hrs ago. No travel or trauma hx. Temp 100 F. Epitrochlear lymph nodes are enlarged. At the distal radial aspect of right upper extremity, an 8 x 13 warm, erythematous area of edema that is tender to palpation is seen. There is a warm, tender red streak extending from this area to the elbow. Organism?

a. H. influenzae
b. B-hemolytic strep
c. psuedomonas aeruginosa
d. salmonella choleraesuis
e. staph epidermidis (INCORRECT)

Follow up of 47 yo M after being diagnosed with stage 1 HTN 5 months ago. HTN has not been controlled despite 3 mo trial of diet modification and exercise. HCTZ therapy was begun. He has felt fatigued over the past month. No abnormalities on noted on exam. Cause of patient's symptoms?

a. hyperglycemia
b. hyperlipidemia
c. hypocalcemia
d. hypokalemia
e. hyponatremia (INCORRECT)

52 yo F, G2P2, 2 yr hx of increasingly frequent loss of urine. She feels an overwhelming urge to void and completely saturates an absorbant pad before reaching the bathroom. The most frequent occurrence of loss of urine is when she steps out of the car. She wakes up 3-4x a night to urinate. No loss of urine with coughing or sneezing. She is being treated with a diuretic for HTN. Vitals are normal. Exam of abdomen, external genitalia,vagina, and cervix are all normal. Uterus and adexna are normal on palpation. No loss of urine with valsalva or coughing. Post void volume is 50 mL. Urinanalysis is normal. Diagnosis?

a. overactive bladder with incontinence
b. overflow incontinence (INCORRECT)
c. stress incontinence
d. UTI
e. vesicovaginal fistula

72 yo M in the ER; has vomited 3x within the last 12 hrs and has also felt dizzy. He has been taking ibuprofen daily for arthritis since 5 yrs. He increased the dose of ibuprofen in the past 4 weeks due to increasing joint pain but he does not recall how many pills he took. Due to his joint pain he is unable to take his daily walks. He does not have chest pain. He had an uncomplicated MI 10 yrs ago. He looks pale. BP 90/60, HR 110, RR 24/min, temp 99.5 F. Dried blood is seen near the mouth. Lungs are clear to auscultation and heart sounds are normal. Stool appears black and is positive for occult blood. Hematocrit is 22% and platelet ct 215,000. Hemorrhagic gastritis with no active site of bleeding is seen on upper GI endoscopy. Patient becomes short of breath 3o minutes after fluid resuscitation with crystalloid solution and transfusion of 4 units of crossmatched packed RBCs. Diffuse rhonchi and crackles are heard bilaterally. Cause of dyspnea?

a. ABO incompatibility
b. Acute resp distress syndrome (INCORRECT)
c. Aspiration of gastric contents
d. Fluid overload
e. pulmonary embolism

43 yo M, 3 mo hx of fatigue, generalized weakness, & depressed mood; has also experienced cold intolerance, muscle aches, & constipation. He is a strict vegetarian. His wife has hypothyroidism treated with levothyroxine. BP 120/95, HR 62/min, temp 98.6 F. He has dry skin. Thyroid is not enlarged and testes are small. His Achilles tendon reflex has a delayed relaxation phase. His TSH is 0.01 microunits/mL, T4 is 2.3 microunits/dL. Mechanism of findings?

a. autoantibodies against thyroglobulin
b. deficient pituitary production of TSH
c. dietary deficiency of iodine
d. increased production of thyroid binding globulin
e. surreptitious levothyroxine use (INCORRECT)

Asymptomatic 37 yo African American M, preemployment exam, with no hx of serious illness and is not on any meds. He is not a smoker and exercises regularly. Both his mother and 40 yo brother have HTN. BMI is 23. HR 82/min, BP 144/92. No other abnormalities. Next step?

a. routine exam in 1 yr
b. decreased dietary intake of sodium (INCORRECT)
c. 24 hr urine collection for measurement of cathecholamines
d. second measurement of blood pressure in two weeks
e. exercise stress test

47 yo M with 2 yr hx of increasing left knee pain. Initially pain was only felt when playing basketball or running, but now he has pain all the time. During the last 6 months his pain has been waking him up at night. Pain is worse with walking or prolonged standing. He used to take part in sports 2-3x weely but now has stopped participating due to pain. When he was 16 he fractured his left proximal tibia while playing football for at which point he underwent open reduction and internal fixation. Says he occasionally has swelling of knee but has not had any locking or catching. There is a varus deformity of left knee on exam. Anterior and posterior drawer tests are are negative. Explanation for these findings?

a. nonunion of his prior fracture
b. patella tendinitis (INCORRECT)
c. post traumatic arthritis
d. tear of anterior cruciate ligament
e. tear of medial meniscus

37 yo F G2P2 with 3 month hx of SOB with exertion with hx of rheumatic fever during childhood. Prenatal exams 5 and 10 yrs ago revealed a murmur, but otherwise both pregnancies were uncomplicated. She takes no meds, and does not smoke or drink alcohol. BMI is 19. BP 105/70, HR 70/min and irregular, RR 18/min, 98.2 F. There is malar flushing and JVD on exam and lungs are clear to auscultation. A grade 2/6 late diastolic murmur is heard at apex. There is a loud S1. Cause of patient's dyspnea?

a. Decreased left atrial pressure
b. Decreases left ventricular pressure
c. Decreased pulm venous pressure
d. Increased left ventricular pressure (INCORRECT)
e. Increased pulm venous pressure

4 yo boy with2 week hx of increasing left ear pain. Temp 101.3 F. Left ear has erythematous, bulging tympanic membrane and an edematous auditory canal with a small amount of mucopurulent discharge. Ear appears to be displaced laterally with moderate tenderness behind the ear. No abnormalities in right ear. A 1 cm nontender, freely mobile mass is palpated over left anterior neck. No abnormalities in pharynx. Next step in managment?

a. monospot test
b. tympanography
c. CT scan of temporal bone
d. hydrocortisone/polymyxin/neomycin ear drops
e. oral amoxicillin therapy (INCORRECT)

47 yo M in ER severe neck pain 2 hrs after lifting a heavy tool at his contruction job. Pain is worse when he turns his neck or coughs and now radiates over his right shoulder an arm. He has a 3 yr hx of intermittent neck pain that is better with ibuprofen. Currently he is in severe distress. Finds it difficult to turn his head; when asked to do so he turns it slowly due to pain. Muscle strength is 4/5 in elbow flexor and wrist extensor muscles on the right. Right biceps and brachoradialis muscles have decreased DTRs. Cause of these findings?

a. central cord syndrome
b. compression of lower brachial plexus
c. compression of musculocutaneous nerve
d. compression upper brachial plexus (INCORRECT)
e. herniated disk at C5-6
 
Members don't see this ad :)
77 yo F, severe SOB after colectomy 3 days ago. Current meds: subcutaneous heparin & patient-controlled morphine. HR 140/min, resp 28/min, temp 98.6 F, BP 110/60, O2 sat: 89%. Breath sounds are reduced on right. Surgical incision as no discharge or erythema. ABGs: pH 7.38, PCO2 23, PO2 55. Atelectasis at both lung bases on CXR.
Nonspecific ST-T wave changes on EKG. Besides O2 theraphy, next step in mangment?

a. Spiral CT scan of chest (check for PE)
b. oral warfarin therapy
c. IV recombinant tPA
d. Pulm angiography (INCORRECT)
e. Placement of IVC filter

37 yo M who returned form a trip to Mexico 1 mo ago, with fever & bilateral flank pain. Takes no meds & has fever (102.2 F). Spleen is palpable 2 cm under coastal margin with deep inspiration. Hb 9, reticulocytes 5%, LDH 300. Next step in Diagnosis?

a. Aspiration of lesion
b. examination of duodenal aspirate for ova & parasites
c. examination of stool for ova & parasites (INCORRECT)
d. serum antibody titer for Entameba histolytica (Mexico, Hepatomegaly, posible amebic liver abscess, Histolycia = increase LDH)
e. serum antibody titer for Plasmodium species
f. serum antibody titer for strongyloides stercoralis
g. serum antibody titer for visceral larva migrans
h. skin test for trichinella spiralis
i. thick and thin blood smears for malaria

77 yo F 24 hr hx of watery diarrhea & nausea is admitted. So far during her stay she has vomited 4x and has not been able to drink or eat. Patient has hx of HTN, DM2, hyperlipidemia, chronic renal insufficiency. She is taking atorvastatin, lisinopril, HCTZ, & aspirin. She has dry mucous membranes. Abdomen is non-tender & soft. Vitals: BP 100/60, HR 110/min, temp 99.3 F, RR 22/min. Labs: Na+ 132, K+ 8.2, Cl- 99, HCO3 18, BUN 95, Creatinine 6.4, Glucose 199. Peaked T waves and QRS of 0.16 on EKG. Next step in managment?

a. administer bicarbonate
b. administer calcium gluconate (Hyperkalemia with EKG Changes, Calcium to stabilize Cardiac membrane, prevent life threatening arrhythmia)
c. administer insulin and 50% dextrose in water
d. administer sodium polystyrene sulfonate (Kayexalate)
e. Schedule dialysis (INCORRECT - I thought she had hyperkalemia and renal failure so that was an indication for dialysis???)

,
23 yo F with 4 days hx of abd discomfort, loss of appetite, nausea. There is diffuse abd tenderness to deep palpation. RR 28/min. Labs: Na+ 132, Cl- 102, K+ 6, HCO3 10, Glucose 450. Cause?

a. addison's disease (INCORRECT)
b. diabetes insipidus
c. diabetic ketoacidosis (increase anion game metabolic acidosis, increase K+ due to decrease insulin, nausea classic for DKA)
d. hemolysis
e. laxative abuse
f. lower GI tract fistula
g. renal tubular acidosis
h. SLE
i. tumor lysis synd
j. vomiting

25 yo F, 4 hr hx of SOB, marked malaise, weakness, & fatigue. 24 hrs ago she had a strenous physical workout follwoed by muscle soreness. She took ibuprofen for her pain 1 hr prior to onset of her symptoms. Has hx of migraines for which she's been metoprolol 2x daily for pro[hylaxis over the past month. Exam: mild periorbital edema & facial flushing. There are multiple wheezes through out lung fields. BP 100/70. Substance to prevent recurrence of similar episode?

a. acetaminophen (INCORRECT)
b. aspirin (Ibuprofen probably caused this reaction, so avoid any NSAIDs, you don't want to shift the arachidonic pathway towards the LKTs)
c. meperidine
d. prednisone
e. propoxyphene

87 yo nursing home resident, with fever for 1 day. She has urinary incontinence for which a catheter was placed 2 weeks ago. Hx of Alzheimers dementia and cannot verbally communicate. Has moist and pink mucous membranes. Vitals: HR 86/min, BP 120/74, resp 14/min, temp 100 F. Urinanalysis: cloudy brown, pH 8.8, blood 2+,
Glucose -ve, protein 2+, RBC: too numerous to count, WBC 20-25, Nitrites 3+, Leukocyte esterase 3+, bacteria: many. Gram -ve bacilli on gram stain. Measure most likely to prevent patient's current condition?

a. use of incontinence briefs instead of catheter (avoid catheterization whenever possible to reduce the possibility of compilations)
b. changing catheter daily (INCORRECT)
c. cleaning urethral orifice with povidone iodide daily
d. flushing the catheter with antibiotic solution daily
e. oral antibiotic prophylaxis
f. oral oxybutynin therapy

57 yo F, hx of progressive cough productive of 2-3 tablespoons of yellow sputum daily for 2 yrs. It lasts all day but it worse in the morning. She is unable to walk more than one block before becoming short of breath. No hx of fever, hemoptysis, weight loss, or night sweats. Smoking hx of 2 packs daily for 40 yrs. BMI 32. BP 150/90, HR 9/min, RR 20/min. Has an O2 saturation of 90%. Breath sounds are decreased in all lung fields with rhonchi heard at bases and a prolonged expiratory phase. She has sinus tachycardia and P pulmonale on EKG. PFTs show FEV1 greater than 80% of predicted, & FEV1/FVC ratio of 70% of predicted and a diffusion lung capacity of carbon monoxide greater than 60 % of predicted. Diagnosis?

a. chronic bronchitis (2 years of cough, had the same feeling with DLCO but I am assuming the just tried to say is 60% of expected, ie. less)
b. cystic fibrosis
c. emphysema (INCORRECT - I thought only emphysema had an increased CO diffusion capacity?)
d kartegener's syndrome
e. post nasal drip syndrome ie.

42 yo M, MVA, undergoes splenectomy and one hour later has severe SOB. Other injuries are left rib and pelvic fractures. Vitals: BP 80/60, HR 133/min, RR 35/min, temp 97.4 F. He has slight distention & absent bowel sounds on abd exam. Next step in management.

a. Xray of chest
b. transfusion type specific uncrossedmatched blood
c. IV heparin therapy
d. fiberoptic bronchoscopy
e. endotracheal intubation (INCORRECT)
f. placement of vena cava filter
g. needle thoracostomy (pt is unstable, Pneumothorax probably after rib fracture, Hypotension, Tachycardia, Tachypnea, decompress with need then chest tube)


72 yo F 3 mo hx of bilateral knee pain with weight bearing. OTC naproxen at maximum dose over past 6 weeks has partially releived the pain. Has Hx of HTN, CAD, CHF, & osteoporosis. Also has hx of stable exertional angina & moderate SOB with exertion. She is taking atenolol, potassium supplements, furosemide. BP 115/70, HR 62/min. She has JVD and lungs are clear to auscultation. CVS exam shows no murmurs or gallops but she has pedal edema. She has crepitus of both knees, and bony prominences at proximal tibial bone on both sides. Labs show potassium of 5.1, BUN of 47 (baseline 20), creatinine of 2.2 (baseline of 1). Which medication is responsible for abnormal lab findings?

a. atenolol
b. furosemide
c. naproxen (naproxen for 6 weeks high does, increase BUN, Cr, NSAID dame the kidney tubules)
d. nitrates
e. potassium supplements (INCORRECT)

67 yo alcoholic male comes for routine exam. Has a 15 yr hx of poorly controlled HTN for which he takes HCTZ but he is not compliant. His blood pressure at the current visit is 170/102 and was at this level for the last 3 office visits. Funduscope findings: AV nicking & tortuosity of the arteries. Patient is at greatest risk for?

a. aortic aneurysm (INCORRECT) (not a smoker)
b. glaucoma
c. myocardial infarction (pt. has risk factors for MI)
d. pulmonary HTN
e. subarachnoid hemorrage

67 yo African American F comes for routine exam. She reached menopause at age 44. She has hypothyroidism and is taking thyroid replacement therapy. Has smoking hx of 1 pack/daily for 45 yrs. BMI 21. Which factor decreases patient's risk for osteoporosis?

a. African American race (increase bone density)
b .cigarette smoking
c. early menopause
d. thin body habitus
e. thyroid replacement therapy (INCORRECT)

37 yo F with known ITP has sudden onset of fever 7 days after undergoing splenectomy. She has had 2 days of mild shoulder pain & moderate abd pain. Hb has been between 9.5 and 10 since the splenectomy. Currently she is on hydrocodone and docusate. She has a fever of 102.4 F. Abd Exam; mild distention & diffuse tenderness without rebound, rigidity, or guarding, and absent bowel sounds. Labs: Hb 9.8, Leukocyte ct. 21,300, segmented neutrophils 50%, bands 17%, eosinophils 3%, lymphocytes 25%, monocytes 5%, platelet ct 105,000, amylase 124. Xray shows left pleurual effusion. Cause of findings?

a. aspiration
b. gastric perforation (INCORRECT) (no air under the diapharm)
c. pancreatitis
d. pneumonia
e. subphrenic abscess (Post-splenectomy subphrenic abscess, phrenic nerve impingement cause refered shoulder pain, abscess fits the Fever, Increase Leukocyte count)

82 yo M, lives in nursing home: SOB at rest, increasing malaise, generalized weakness, & depressed mood over past 6 weeks. He bruises easily and has bleeding & infla)mmation of the gums. He reports a minor skin abrasion 1 month ago that has not healed completely, and he appears chronically ill. Exam: numerous petechiae, eccymosis, & hypertrophic lichenified patches over upper extremities. Mild pedal edema is present. He is oriented to TPP and has no short or long term memory loss. Labs including CBC, urinanalyis, measurement of serum electrolyte concentrations, are all with in normal limits. Diagnosis?

a. Acquired factor VIII deficiency (INCORRECT)
b. hypersensitivity vasculitis
c. polymyalgia rheumatica
d. psoriasis
e. thrombocytopenic purpura
f. vit B12 deficiency
g. vit C deficiency (Scurvy, malnutrition, easy bruising, bleeding gums)

Asymptomatic 72 yo F for routine exam, takes no meds. DEXA findings at age 65 were normal. At her last visit 1 yr ago pap smear, mammogrpahy, sigmoidoscopy, were all normal. Her serum cholesterol was 190 and glucose was 80. She remains sexually active and does not smoke. She drinks 1-2 glasses of wine 3x weekly. BMI 23. BP 140/85. Physical and pelvic exams show no abnormalities. Next step in managment?

a. measurment of fasting glucose concentration
b. serum lipid studies (INCORRECT)
c. ECG
d. Influenza virus vaccination
e. sigmoidoscopy

72 yo lethargic appearing dementia patient who lives in a nursing home brought to ER 2 days after onset of fever, confusion, and generalized weakness. Symptoms began following irrigation of his urinary catheter, which wasn't draining well. He has hx of COPD and osteoarthritis. Vitals: Temp 97.2 F, HR 110, BP 90/50. Has supple neck on exam. Herbenden nodes are seen on the hands. Breath sounds are decreased throughout lungs with no dullness. Cranial nerves are intact and DTRs are normal. Labs: Hct 50%, leukocyte ct 15,400, segmented neutrophils 74%, bands 4%, lymphocytes 22%, Na+ 134, K+ 4.2, Cl- 86, HCO3 24, BUN 12, creatinine 1.1. Sinus tachycardiea and some flattening of ST-T segments. Most likely explanation for hypotension?

a. decreased cardiac contractility
b. decreased systemic vascular resistance (Septic shock secondary to UTI)
c. deficiency of mineralcorticoids
d. impaired vagal reflexes
e. impaired venous return (INCORRECT)

Follow up of a 6 mo boy who's had chronic constipation since age 1 week. Current treatment with rectal stimulation, glycerin suppositories, & 4 oz of prune juice produces one firm string like stool every 4 days. After being breast fed until 2 weeks of age he was switched to cow's milk-based formula. At the moment his diet also consists of cereals, fruits, pureed vegetables. Abd exam: distention with no tenderness. Rectal exam shows no palpable stool in ampulla. Next step in management?

a. switch to lactose-free formula
b. test of stool for botulism toxin (INCORRECT)
c. stool culture
d. measurement of serum TSH concentration
e. rectal manometry (Failure to pass Meconium in a neonate is Hirshprung dz until proven otherwise, do Manometry)
f. upper GI series

52 yo F, routine exam. She was started on daily hormone replacment therapy (estradiol and medroxyprogesterone) one month ago to treat severe vasomotor symptoms. But she has been taking the medroxyprogesterone intermittently due to severe depression and mood changes while on it. For the first 6 mo of the regimen menses had stopped. However, since she changed her regimen she has vaginal spotting 2 -3x/ monthly. Next step in managment?

a. discontinue medroxyprogesterone therapy only
b. increase dosage of estradiol
c. add an antidepressant to the medication regimen (INCORRECT)
d. endometrial abalation
e. endometrial biopsy (vaginal spoting after HRT (1+mo) can be Endometrial Hyperplasia you need to Biopsy to rule out pre-malignany or malignancy)


3 yo girl, 1 day hx of fever and ear pain, has had clear nasal discharge and cough for 3 days. Over last 12 months she has had a hx of several ear infections and one episode of strep pharangitis. Her family has two cats and the father smokes in the home. She frequently swims in the family's swimming pool. BP 80/50, HR 110/min, RR 22/min, temp 101.3 F. She has clear nasal discharage, erythemea and bulging of right tympanic membrane and erythema of throat without exudate. Lungs are clear toa, auscultation. Most appropriate recommendation to prevent recurrence of this patient's condition?

a. avoidance of passive smoke exposure (research this one, not completely sure)
b. removal of cats from the house
c. use of earplugs while swimming (INCORRECT)
d. daily otic antibiotics
e. prophylactic oral decongestants
f. tonsilectomy

67 yo F, 2 mo hx of intermittent burning sensation in her epigastrium. It presents after she walks one block or climbs one flight of stairs and is sometimes associated with lightheadedness, and profuse sweating. Epigastric discomfort is better with rest and is not relieved by antacids. Her meds include daily multivitamin and calcium supplements. Smoker for 40 yrs with 1 pack/daily. She has one drink of alcohol 1-2x/ weekly. BP 150/90, HR 98/min, resp 15/min, temp 98.6F. Results of labs, including cardiac enzymes, are within normal range. No abnormalities are noted on EKG. Most appropriate next step in diagnosis?

a. ambulatory EKG monitoring (INCORRECT)
b. abdominal U/S
c. Cardiac stress schintography (riks factors for CAD, epigastric=vague way of saying substernal, stable angina, do exercise to reproduce the ischemia check EKG for any ST segment changes)
d. 24 hr monitoring of esophageal pH
e. esophagogastroduodenoscopy

62 yo M brought 4 hrs after the sudden onset of vomiting & severe abd pain. Appears diaphoretic and in acute distress. He has had mild,intermittent epigastric pain over past year that is relieved by antacids. No hx of serious illness. He is a smoker : 1 pack/daily for 35 yrs and has 1-2 drinks/ daily. BP 115/75, HR 100/min, RR 18/min, temp 101.1 F. Abd is rigid and tympanic to percussion over the liver, with severe diffuse tenderness. No bowel sounds are present. Next step in diagnosis?

a. X-rays of chest and abdomen (severe pain, pain over past year relieve by anti acids=gastritis, this on is probably a perforation)
b. barium enema
c. water soluble contrast swallow
d. CT scan of abd (INCORRECT)
e. colonoscopy
f. flexible esophagogastroduodenoscopy

32 yo F, 2 day hx of fever & an area of swelling & redness on her right forearm. She noticed red streaks extending from her forearm to her elbow 6 hrs ago. No travel or trauma hx. Temp 100 F. Epitrochlear lymph nodes are enlarged. At the distal radial aspect of right upper extremity, an 8 x 13 warm, erythematous area of edema that is tender to palpation is seen. There is a warm, tender red streak extending from this area to the elbow. Organism?

a. H. influenzae
b. B-hemolytic strep (cellulitis)
c. psuedomonas aeruginosa
d. salmonella choleraesuis
e. staph epidermidis (INCORRECT) (no trauma Hx or prosthetic divice)

Follow up of 47 yo M after being diagnosed with stage 1 HTN 5 months ago. HTN has not been controlled despite 3 mo trial of diet modification and exercise. HCTZ therapy was begun. He has felt fatigued over the past month. No abnormalities on noted on exam. Cause of patient's symptoms?

a. hyperglycemia
b. hyperlipidemia
c. hypocalcemia
d. hypokalemia (weakness)
e. hyponatremia (INCORRECT) (this will cause CNS problems)

52 yo F, G2P2, 2 yr hx of increasingly frequent loss of urine. She feels an overwhelming urge to void and completely saturates an absorbant pad before reaching the bathroom. The most frequent occurrence of loss of urine is when she steps out of the car. She wakes up 3-4x a night to urinate. No loss of urine with coughing or sneezing. She is being treated with a diuretic for HTN. Vitals are normal. Exam of abdomen, external genitalia,vagina, and cervix are all normal. Uterus and adexna are normal on palpation. No loss of urine with valsalva or coughing. Post void volume is 50 mL. Urinanalysis is normal. Diagnosis?

a. overactive bladder with incontinence (urge incontinence)
b. overflow incontinence (INCORRECT)
c. stress incontinence
d. UTI
e. vesicovaginal fistula

72 yo M in the ER; has vomited 3x within the last 12 hrs and has also felt dizzy. He has been taking ibuprofen daily for arthritis since 5 yrs. He increased the dose of ibuprofen in the past 4 weeks due to increasing joint pain but he does not recall how many pills he took. Due to his joint pain he is unable to take his daily walks. He does not have chest pain. He had an uncomplicated MI 10 yrs ago. He looks pale. BP 90/60, HR 110, RR 24/min, temp 99.5 F. Dried blood is seen near the mouth. Lungs are clear to auscultation and heart sounds are normal. Stool appears black and is positive for occult blood. Hematocrit is 22% and platelet ct 215,000. Hemorrhagic gastritis with no active site of bleeding is seen on upper GI endoscopy. Patient becomes short of breath 3o minutes after fluid resuscitation with crystalloid solution and transfusion of 4 units of crossmatched packed RBCs. Diffuse rhonchi and crackles are heard bilaterally. Cause of dyspnea?

a. ABO incompatibility
b. Acute resp distress syndrome (INCORRECT)
c. Aspiration of gastric contents
d. Fluid overload (pulmonary edema due to excessive fluid resuscitation)
e. pulmonary embolism

43 yo M, 3 mo hx of fatigue, generalized weakness, & depressed mood; has also experienced cold intolerance, muscle aches, & constipation. He is a strict vegetarian. His wife has hypothyroidism treated with levothyroxine. BP 120/95, HR 62/min, temp 98.6 F. He has dry skin. Thyroid is not enlarged and testes are small. His Achilles tendon reflex has a delayed relaxation phase. His TSH is 0.01 microunits/mL, T4 is 2.3 microunits/dL. Mechanism of findings?

a. autoantibodies against thyroglobulin
b. deficient pituitary production of TSH (hypothyroidism, Pituitary defect)
c. dietary deficiency of iodine
d. increased production of thyroid binding globulin
e. surreptitious levothyroxine use (INCORRECT)

Asymptomatic 37 yo African American M, preemployment exam, with no hx of serious illness and is not on any meds. He is not a smoker and exercises regularly. Both his mother and 40 yo brother have HTN. BMI is 23. HR 82/min, BP 144/92. No other abnormalities. Next step?

a. routine exam in 1 yr
b. decreased dietary intake of sodium (INCORRECT)
c. 24 hr urine collection for measurement of cathecholamines
d. second measurement of blood pressure in two weeks (Two separate measurements are needed to Dx HTN)
e. exercise stress test

47 yo M with 2 yr hx of increasing left knee pain. Initially pain was only felt when playing basketball or running, but now he has pain all the time. During the last 6 months his pain has been waking him up at night. Pain is worse with walking or prolonged standing. He used to take part in sports 2-3x weely but now has stopped participating due to pain. When he was 16 he fractured his left proximal tibia while playing football for at which point he underwent open reduction and internal fixation. Says he occasionally has swelling of knee but has not had any locking or catching. There is a varus deformity of left knee on exam. Anterior and posterior drawer tests are are negative. Explanation for these findings?

a. nonunion of his prior fracture
b. patella tendinitis (INCORRECT)
c. post traumatic arthritis (fracture when he was 16yrs old, probably developed scar tissue and that accelerated arthritis)
d. tear of anterior cruciate ligament
e. tear of medial meniscus

37 yo F G2P2 with 3 month hx of SOB with exertion with hx of rheumatic fever during childhood. Prenatal exams 5 and 10 yrs ago revealed a murmur, but otherwise both pregnancies were uncomplicated. She takes no meds, and does not smoke or drink alcohol. BMI is 19. BP 105/70, HR 70/min and irregular, RR 18/min, 98.2 F. There is malar flushing and JVD on exam and lungs are clear to auscultation. A grade 2/6 late diastolic murmur is heard at apex. There is a loud S1. Cause of patient's dyspnea?

a. Decreased left atrial pressure
b. Decreases left ventricular pressure
c. Decreased pulm venous pressure
d. Increased left ventricular pressure (INCORRECT)
e. Increased pull venous pressure (Mitral stenosis is very common in pregnancy due to increase volume, she has the murmur in apex, loud S1, this cause increase Left atrial pressure, which causes back up of blood though the pulmonary Veins)

4 yo boy with2 week hx of increasing left ear pain. Temp 101.3 F. Left ear has erythematous, bulging tympanic membrane and an edematous auditory canal with a small amount of mucopurulent discharge. Ear appears to be displaced laterally with moderate tenderness behind the ear. No abnormalities in right ear. A 1 cm nontender, freely mobile mass is palpated over left anterior neck. No abnormalities in pharynx. Next step in managment?

a. monospot test
b. tympanography
c. CT scan of temporal bone (ear displace laterally, post auricular tenderness= Mastoiditis,which is is a complication of AOM, check temporal bone with CT)
d. hydrocortisone/polymyxin/neomycin ear drops
e. oral amoxicillin therapy (INCORRECT)

47 yo M in ER severe neck pain 2 hrs after lifting a heavy tool at his contruction job. Pain is worse when he turns his neck or coughs and now radiates over his right shoulder an arm. He has a 3 yr hx of intermittent neck pain that is better with ibuprofen. Currently he is in severe distress. Finds it difficult to turn his head; when asked to do so he turns it slowly due to pain. Muscle strength is 4/5 in elbow flexor and wrist extensor muscles on the right. Right biceps and brachoradialis muscles have decreased DTRs. Cause of these findings?

a. central cord syndrome
b. compression of lower brachial plexus
c. compression of musculocutaneous nerve
d. compression upper brachial plexus (INCORRECT)
e. herniated disk at C5-6

hope it helps
 
I have some more questions. I'm sorry I know its a lot. I couldn't find any other resources with answers, and I am really confused about these questions. I would be extremely grateful if I could get help with them. It would be great if you guys could help with what ever questions you know, even if its not all of them. Thanks in advance.

I'm going to take a stab at a few. I will highlight in bold and explain the reasoning behind it. Please make sure you double-check the explanations and answers I select to ensure that I am not giving you the wrong information.


77 yo F, severe SOB after colectomy 3 days ago. Current meds: subcutaneous heparin & patient-controlled morphine. HR 140/min, resp 28/min, temp 98.6 F, BP 110/60, O2 sat: 89%. Breath sounds are reduced on right. Surgical incision as no discharge or erythema. ABGs: pH 7.38, PCO2 23, PO2 55. Atelectasis at both lung bases on CXR.
Nonspecific ST-T wave changes on EKG. Besides O2 theraphy, next step in mangment?

a. Spiral CT scan of chest (I believe this is the correct answer because it appears she either has a PE or a consolidation of some sort and you want to find out what it could be so this is the most likely next step. The one you chose is more invasive and wouldn't be necessary at this point based on ABG and other factors)
b. oral warfarin therapy
c. IV recombinant tPA
d. Pulm angiography (INCORRECT)
e. Placement of IVC filter

37 yo M who returned form a trip to Mexico 1 mo ago, with fever & bilateral flank pain. Takes no meds & has fever (102.2 F). Spleen is palpable 2 cm under coastal margin with deep inspiration. Hb 9, reticulocytes 5%, LDH 300. Next step in Diagnosis? In this particular case the guys LDH is elevated indicating a hemolysis of some sort, thus explaining the increase in reticulocytes. Thus, you would have to find one that causes hemolysis while taking into account he returned from a trip to Mexico.My money would be on either B or D. Im sorry I cannot be much help on this one

a. Aspiration of lesion
b. examination of duodenal aspirate for ova & parasites
c. examination of stool for ova & parasites (INCORRECT)
d. serum antibody titer for Entameba histolytica
e. serum antibody titer for Plasmodium species
f. serum antibody titer for strongyloides stercoralis
g. serum antibody titer for visceral larva migrans
h. skin test for trichinella spiralis
i. thick and thin blood smears for malaria

77 yo F 24 hr hx of watery diarrhea & nausea is admitted. So far during her stay she has vomited 4x and has not been able to drink or eat. Patient has hx of HTN, DM2, hyperlipidemia, chronic renal insufficiency. She is taking atorvastatin, lisinopril, HCTZ, & aspirin. She has dry mucous membranes. Abdomen is non-tender & soft. Vitals: BP 100/60, HR 110/min, temp 99.3 F, RR 22/min. Labs: Na+ 132, K+ 8.2, Cl- 99, HCO3 18, BUN 95, Creatinine 6.4, Glucose 199. Peaked T waves and QRS of 0.16 on EKG. Next step in managment? In this particular case you will notice that her potassium level is EXTREMELY elevated (normal K values can range from 3.5-4.5, maybe slightly higher, depending on the specific lab protocol). Thus, since she is having EKG abnormalities the first thing you want to do is stabilize the heart by using calcium then you can correct elevated serum K by administering insulin or giving bicarb so that it can migrate into the cell, then you can further correct excess K by administering sodium polystyrene and/or loop diuretic (if they are having renal dysfunction or in ESRD loops will not work so sodium polystyrene will be of most benefit). Indication for renal dialysis would follow the vowels AEIOU (Acidosis, Electrolyte disturbance, Ingested toxins, Overload of fluid, Uremia).

a. administer bicarbonate
b. administer calcium gluconate
c. administer insulin and 50% dextrose in water
d. administer sodium polystyrene sulfonate (Kayexalate)
e. Schedule dialysis (INCORRECT - I thought she had hyperkalemia and renal failure so that was an indication for dialysis???)
,
23 yo F with 4 days hx of abd discomfort, loss of appetite, nausea. There is diffuse abd tenderness to deep palpation. RR 28/min. Labs: Na+ 132, Cl- 102, K+ 6, HCO3 10, Glucose 450. Cause? In this particular case, based off lab values, I would have to say it appears she is undergoing a DKA due to an elevated blood glucose, elevated potassium level, elevated respiratory rate, and decreased bicarb). To confirm this I calculated the anion gap ((Na-(CL+HCO3)) which gave me a value of 19 (normal values range between 5-12).

a. addison's disease (INCORRECT)
b. diabetes insipidus
c. diabetic ketoacidosis
d. hemolysis
e. laxative abuse
f. lower GI tract fistula
g. renal tubular acidosis
h. SLE
i. tumor lysis synd
j. vomiting

25 yo F, 4 hr hx of SOB, marked malaise, weakness, & fatigue. 24 hrs ago she had a strenous physical workout follwoed by muscle soreness. She took ibuprofen for her pain 1 hr prior to onset of her symptoms. Has hx of migraines for which she's been metoprolol 2x daily for pro[hylaxis over the past month. Exam: mild periorbital edema & facial flushing. There are multiple wheezes through out lung fields. BP 100/70. Substance to prevent recurrence of similar episode? I am not completely sure about this but it appears she may have a case of exercise-induced asthma with possible acute kidney injury from NSAID (NSAID blocks synthesis of prostaglandin causing vasoconstricition of afferent renal arteriole). Im sorry I cannot help more

a. acetaminophen (INCORRECT)
b. aspirin
c. meperidine
d. prednisone
e. propoxyphene

87 yo nursing home resident, with fever for 1 day. She has urinary incontinence for which a catheter was placed 2 weeks ago. Hx of Alzheimers dementia and cannot verbally communicate. Has moist and pink mucous membranes. Vitals: HR 86/min, BP 120/74, resp 14/min, temp 100 F. Urinanalysis: cloudy brown, pH 8.8, blood 2+,
Glucose -ve, protein 2+, RBC: too numerous to count, WBC 20-25, Nitrites 3+, Leukocyte esterase 3+, bacteria: many. Gram -ve bacilli on gram stain. Measure most likely to prevent patient's current condition? This appears to be one of those "safety science" questions. If they would have given a choice where the cath could be replaced weekly I would have gone with that. However, since they didn't I guess the only one that appears correct in preventing the patients current condition is one that didnt use a cath at all so I would probably go with A. I could be wrong though

a. use of incontinence briefs instead of catheter
b. changing catheter daily (INCORRECT)
c. cleaning urethral orifice with povidone iodide daily
d. flushing the catheter with antibiotic solution daily
e. oral antibiotic prophylaxis
f. oral oxybutynin therapy

57 yo F, hx of progressive cough productive of 2-3 tablespoons of yellow sputum daily for 2 yrs. It lasts all day but it worse in the morning. She is unable to walk more than one block before becoming short of breath. No hx of fever, hemoptysis, weight loss, or night sweats. Smoking hx of 2 packs daily for 40 yrs. BMI 32. BP 150/90, HR 96/min, RR 20/min. Has an O2 saturation of 90%. Breath sounds are decreased in all lung fields with rhonchi heard at bases and a prolonged expiratory phase. She has sinus tachycardia and P pulmonale on EKG. PFTs show FEV1 greater than 80% of predicted, & FEV1/FVC ratio of 70% of predicted and a diffusion lung capacity of carbon monoxide greater than 60 % of predicted. Diagnosis? This particular question was rather tricky. The most important pieces of information include 2-3 tablespoons of yellow sputum daily for 2 yrs, Has an O2 saturation of 90%, prolonged expiratory phase, and FEV1 greater than 80% of predicted, & FEV1/FVC ratio of 70% of predicted and a diffusion lung capacity of carbon monoxide greater than 60 % of predicted. All of this indicate, at least in my opinion, a bronchitis. Thus, I am inclined to say that A is the correct answer.

a. chronic bronchitis
b. cystic fibrosis
c. emphysema (INCORRECT - I thought only emphysema had an increased CO diffusion capacity?)
d kartegener's syndrome
e. post nasal drip syndrome

42 yo M, MVA, undergoes splenectomy and one hour later has severe SOB. Other injuries are left rib and pelvic fractures. Vitals: BP 80/60, HR 133/min, RR 35/min, temp 97.4 F. He has slight distention & absent bowel sounds on abd exam. Next step in management. Im not completely sure about this question.

a. Xray of chest
b. transfusion type specific uncrossedmatched blood
c. IV heparin therapy
d. fiberoptic bronchoscopy
e. endotracheal intubation (INCORRECT)
f. placement of vena cava filter
g. needle thoracostomy


72 yo F 3 mo hx of bilateral knee pain with weight bearing. OTC naproxen at maximum dose over past 6 weeks has partially releived the pain. Has Hx of HTN, CAD, CHF, & osteoporosis. Also has hx of stable exertional angina & moderate SOB with exertion. She is taking atenolol, potassium supplements, furosemide. BP 115/70, HR 62/min. She has JVD and lungs are clear to auscultation. CVS exam shows no murmurs or gallops but she has pedal edema. She has crepitus of both knees, and bony prominences at proximal tibial bone on both sides. Labs show potassium of 5.1, BUN of 47 (baseline 20), creatinine of 2.2 (baseline of 1). Which medication is responsible for abnormal lab findings? This question is basically asking which one of the following drugs caused acute kidney injury (AKI) based on the uptrend in creatinine. Since NSAID blocks synthesis of prostaglandin causing vasoconstricition of afferent renal arteriole, and hes been using "naproxen at maximum dose over past 6 weeks" I would suspect that would be the culprit of AKI.

a. atenolol
b. furosemide
c. naproxen
d. nitrates
e. potassium supplements (INCORRECT)

67 yo alcoholic male comes for routine exam. Has a 15 yr hx of poorly controlled HTN for which he takes HCTZ but he is not compliant. His blood pressure at the current visit is 170/102 and was at this level for the last 3 office visits. Funduscope findings: AV nicking & tortuosity of the arteries. Patient is at greatest risk for? I have no clue

a. aortic aneurysm (INCORRECT)
b. glaucoma
c. myocardial infarction
d. pulmonary HTN
e. subarachnoid hemorrage

67 yo African American F comes for routine exam. She reached menopause at age 44. She has hypothyroidism and is taking thyroid replacement therapy. Has smoking hx of 1 pack/daily for 45 yrs. BMI 21. Which factor decreases patient's risk for osteoporosis? You have to know a little background history in genetics to be able to answer this question. It has been proven that African American women have greater bone density when compared to Caucasian women, which decreases their risk for osteoporosis. Thus, the most correct answer is A

a. African American race
b .cigarette smoking
c. early menopause
d. thin body habitus
e. thyroid replacement therapy (INCORRECT)

37 yo F with known ITP has sudden onset of fever 7 days after undergoing splenectomy. She has had 2 days of mild shoulder pain & moderate abd pain. Hb has been between 9.5 and 10 since the splenectomy. Currently she is on hydrocodone and docusate. She has a fever of 102.4 F. Abd Exam; mild distention & diffuse tenderness without rebound, rigidity, or guarding, and absent bowel sounds. Labs: Hb 9.8, Leukocyte ct. 21,300, segmented neutrophils 50%, bands 17%, eosinophils 3%, lymphocytes 25%, monocytes 5%, platelet ct 105,000, amylase 124. Xray shows left pleurual effusion. Cause of findings? I have no clue

a. aspiration
b. gastric perforation (INCORRECT)
c. pancreatitis
d. pneumonia
e. subphrenic abscess

82 yo M, lives in nursing home: SOB at rest, increasing malaise, generalized weakness, & depressed mood over past 6 weeks. He bruises easily and has bleeding & inflammation of the gums. He reports a minor skin abrasion 1 month ago that has not healed completely, and he appears chronically ill. Exam: numerous petechiae, eccymosis, & hypertrophic lichenified patches over upper extremities. Mild pedal edema is present. He is oriented to TPP and has no short or long term memory loss. Labs including CBC, urinanalyis, measurement of serum electrolyte concentrations, are all with in normal limits. Diagnosis? Based on the presentation, I would conclude that this old guy has a vitamin C. I could be wrong but my gut is telling me that I am right

a. Acquired factor VIII deficiency (INCORRECT)
b. hypersensitivity vasculitis
c. polymyalgia rheumatica
d. psoriasis
e. thrombocytopenic purpura
f. vit B12 deficiency
g. vit C deficiency

Asymptomatic 72 yo F for routine exam, takes no meds. DEXA findings at age 65 were normal. At her last visit 1 yr ago pap smear, mammogrpahy, sigmoidoscopy, were all normal. Her serum cholesterol was 190 and glucose was 80. She remains sexually active and does not smoke. She drinks 1-2 glasses of wine 3x weekly. BMI 23. BP 140/85. Physical and pelvic exams show no abnormalities. Next step in managment? I would be inclined to say the next step of management would be to administer a influenza vaccine because as we age immunity wanes. Also, based off all her other test and labs everything is well so the only thing that is left is to vaccinate.

a. measurment of fasting glucose concentration
b. serum lipid studies (INCORRECT)
c. ECG
d. Influenza virus vaccination
e. sigmoidoscopy

72 yo lethargic appearing dementia patient who lives in a nursing home brought to ER 2 days after onset of fever, confusion, and generalized weakness. Symptoms began following irrigation of his urinary catheter, which wasn't draining well. He has hx of COPD and osteoarthritis. Vitals: Temp 97.2 F, HR 110, BP 90/50. Has supple neck on exam. Herbenden nodes are seen on the hands. Breath sounds are decreased throughout lungs with no dullness. Cranial nerves are intact and DTRs are normal. Labs: Hct 50%, leukocyte ct 15,400, segmented neutrophils 74%, bands 4%, lymphocytes 22%, Na+ 134, K+ 4.2, Cl- 86, HCO3 24, BUN 12, creatinine 1.1. Sinus tachycardiea and some flattening of ST-T segments. Most likely explanation for hypotension? It seems like this old person is experiencing a possible UTI since he has increased neutrophilia accompanied by fever, confusion, and generalized weakness. If thats the case, you will have a decreased systemic vascular resistance due to septic shock (Increased HR, decreased SVR, and decreased PCWP) which causing pooling of blood in the venous side.

a. decreased cardiac contractility
b. decreased systemic vascular resistance
c. deficiency of mineralcorticoids
d. impaired vagal reflexes
e. impaired venous return (INCORRECT)

Follow up of a 6 mo boy who's had chronic constipation since age 1 week. Current treatment with rectal stimulation, glycerin suppositories, & 4 oz of prune juice produces one firm string like stool every 4 days. After being breast fed until 2 weeks of age he was switched to cow's milk-based formula. At the moment his diet also consists of cereals, fruits, pureed vegetables. Abd exam: distention with no tenderness. Rectal exam shows no palpable stool in ampulla. Next step in management? This one is tough. However, the clue about not having palpable stool in the amuplla indicates to be there is a stricture that could be indicative of something like hirschsprung.

a. switch to lactose-free formula
b. test of stool for botulism toxin (INCORRECT)
c. stool culture
d. measurement of serum TSH concentration
e. rectal manometry
f. upper GI series

52 yo F, routine exam. She was started on daily hormone replacment therapy (estradiol and medroxyprogesterone) one month ago to treat severe vasomotor symptoms. But she has been taking the medroxyprogesterone intermittently due to severe depression and mood changes while on it. For the first 6 mo of the regimen menses had stopped. However, since she changed her regimen she has vaginal spotting 2 -3x/ monthly. Next step in managment? In this question it is basically asking what you would do next in a patient that was compliant but then became noncompliant with the medication regimen. If you remember, you give both estrogen and progesterone to women undergoing menopause to curb vasomotor symptoms (hot flashes). Since she stopped taking the progesterone, you would probably want to get a biopsy to see if there was metaplasia that could have taken place so I would probably go with E. But, I would double check that answer because I am not completely sure.

a. discontinue medroxyprogesterone therapy only
b. increase dosage of estradiol
c. add an antidepressant to the medication regimen (INCORRECT)
d. endometrial abalation
e. endometrial biopsy

3 yo girl, 1 day hx of fever and ear pain, has had clear nasal discharge and cough for 3 days. Over last 12 months she has had a hx of several ear infections and one episode of strep pharangitis. Her family has two cats and the father smokes in the home. She frequently swims in the family's swimming pool. BP 80/50, HR 110/min, RR 22/min, temp 101.3 F. She has clear nasal discharage, erythemea and bulging of right tympanic membrane and erythema of throat without exudate. Lungs are clear to auscultation. Most appropriate recommendation to prevent recurrence of this patient's condition? This particular question seems to deal with a child who appears to be "immunosuppressed" or having described symptoms because of a secondary cause. If I had to guess, the most probable choices would be A or F. I am not too sure

a. avoidance of passive smoke exposure
b. removal of cats from the house
c. use of earplugs while swimming (INCORRECT)
d. daily otic antibiotics
e. prophylactic oral decongestants
f. tonsilectomy

67 yo F, 2 mo hx of intermittent burning sensation in her epigastrium. It presents after she walks one block or climbs one flight of stairs and is sometimes associated with lightheadedness, and profuse sweating. Epigastric discomfort is better with rest and is not relieved by antacids. Her meds include daily multivitamin and calcium supplements. Smoker for 40 yrs with 1 pack/daily. She has one drink of alcohol 1-2x/ weekly. BP 150/90, HR 98/min, resp 15/min, temp 98.6F. Results of labs, including cardiac enzymes, are within normal range. No abnormalities are noted on EKG. Most appropriate next step in diagnosis? The basis of this question is trying to see if you can pick up on the comorbidities contributing to this female manifestations. However, since the EKG was normal the next best step in diagnosis would have to be a Stress Test (aka cardiac scintography). Also, this may be slightly more advanced than the knowledge tested on step 2 but based off TIMI score and manifestations, some doctors would skip stress test and go straight to an angiography with possible stent placement.

a. ambulatory EKG monitoring (INCORRECT)
b. abdominal U/S
c. Cardiac stress schintography
d. 24 hr monitoring of esophageal pH
e. esophagogastroduodenoscopy

62 yo M brought 4 hrs after the sudden onset of vomiting & severe abd pain. Appears diaphoretic and in acute distress. He has had mild,intermittent epigastric pain over past year that is relieved by antacids. No hx of serious illness. He is a smoker : 1 pack/daily for 35 yrs and has 1-2 drinks/ daily. BP 115/75, HR 100/min, RR 18/min, temp 101.1 F. Abd is rigid and tympanic to percussion over the liver, with severe diffuse tenderness. No bowel sounds are present. Next step in diagnosis? In this particular cause you would probably want to get an x-ray of the chest and abdomen in order to rule in and/or rule out causes. Not to mention, the CT isnt warranted at this point but would probably be necessary later on IF you pick up something weird on x-ray

a. Xrays of chest and abdomen
b. barium enema
c. water soluble contrast swallow
d. CT scan of abd (INCORRECT)
e. colonoscopy
f. flexible esophagogastroduodenoscopy

32 yo F, 2 day hx of fever & an area of swelling & redness on her right forearm. She noticed red streaks extending from her forearm to her elbow 6 hrs ago. No travel or trauma hx. Temp 100 F. Epitrochlear lymph nodes are enlarged. At the distal radial aspect of right upper extremity, an 8 x 13 warm, erythematous area of edema that is tender to palpation is seen. There is a warm, tender red streak extending from this area to the elbow. Organism? For this question you would have to remember which organisms tend to colonize the skin and what manifestations would happen if it were able to produce an infection. However, it appears she may have cellulitis so I would be willing to say the answer is B

a. H. influenzae
b. B-hemolytic strep
c. psuedomonas aeruginosa
d. salmonella choleraesuis
e. staph epidermidis (INCORRECT)

Follow up of 47 yo M after being diagnosed with stage 1 HTN 5 months ago. HTN has not been controlled despite 3 mo trial of diet modification and exercise. HCTZ therapy was begun. He has felt fatigued over the past month. No abnormalities on noted on exam. Cause of patient's symptoms? This one is a little tricky but you have to remember that people with hypertension tend to have "constant" activation of the renin-angiotension-aldosterone system. Thus, it would be same to assume there is an elevation of aldosterone, which tends to retain sodium while kicking out potassium. Thus, I would be inclined to say the answer is D. Double check this reasoning though

a. hyperglycemia
b. hyperlipidemia
c. hypocalcemia
d. hypokalemia
e. hyponatremia (INCORRECT)

52 yo F, G2P2, 2 yr hx of increasingly frequent loss of urine. She feels an overwhelming urge to void and completely saturates an absorbant pad before reaching the bathroom. The most frequent occurrence of loss of urine is when she steps out of the car. She wakes up 3-4x a night to urinate. No loss of urine with coughing or sneezing. She is being treated with a diuretic for HTN. Vitals are normal. Exam of abdomen, external genitalia,vagina, and cervix are all normal. Uterus and adexna are normal on palpation. No loss of urine with valsalva or coughing. Post void volume is 50 mL. Urinanalysis is normal. Diagnosis? This question is somewhat tricky because of the presentations. What gives you a clue into the proper diagnosis is "She wakes up 3-4x a night to urinate" and "No loss of urine with valsalva or coughing." Thus, the only logical choice would be something of an overactive bladder. Just double check to ensure the reasoning is correct

a. overactive bladder with incontinence
b. overflow incontinence (INCORRECT)
c. stress incontinence
d. UTI
e. vesicovaginal fistula

72 yo M in the ER; has vomited 3x within the last 12 hrs and has also felt dizzy. He has been taking ibuprofen daily for arthritis since 5 yrs. He increased the dose of ibuprofen in the past 4 weeks due to increasing joint pain but he does not recall how many pills he took. Due to his joint pain he is unable to take his daily walks. He does not have chest pain. He had an uncomplicated MI 10 yrs ago. He looks pale. BP 90/60, HR 110, RR 24/min, temp 99.5 F. Dried blood is seen near the mouth. Lungs are clear to auscultation and heart sounds are normal. Stool appears black and is positive for occult blood. Hematocrit is 22% and platelet ct 215,000. Hemorrhagic gastritis with no active site of bleeding is seen on upper GI endoscopy. Patient becomes short of breath 3o minutes after fluid resuscitation with crystalloid solution and transfusion of 4 units of crossmatched packed RBCs. Diffuse rhonchi and crackles are heard bilaterally. Cause of dyspnea? If you look at the presentation the guy is all sorts of messed up. What stands out the most is how his "Lungs are clear to auscultation and heart sounds are normal" PRIOR to "fluid resuscitation with crystalloid solution and transfusion of 4 units of crossmatched packed RBCs."

a. ABO incompatibility
b. Acute resp distress syndrome (INCORRECT)
c. Aspiration of gastric contents
d. Fluid overload
e. pulmonary embolism

43 yo M, 3 mo hx of fatigue, generalized weakness, & depressed mood; has also experienced cold intolerance, muscle aches, & constipation. He is a strict vegetarian. His wife has hypothyroidism treated with levothyroxine. BP 120/95, HR 62/min, temp 98.6 F. He has dry skin. Thyroid is not enlarged and testes are small. His Achilles tendon reflex has a delayed relaxation phase. His TSH is 0.01 microunits/mL, T4 is 2.3 microunits/dL. Mechanism of findings? His feelings of fatigue, generalized weakness, and depressed mood based of this question would indicate a HYPOthyroidim. Now, you have to see which answer choice best fits the scenario.

a. autoantibodies against thyroglobulin
b. deficient pituitary production of TSH
c. dietary deficiency of iodine
d. increased production of thyroid binding globulin
e. surreptitious levothyroxine use (INCORRECT)

Asymptomatic 37 yo African American M, preemployment exam, with no hx of serious illness and is not on any meds. He is not a smoker and exercises regularly. Both his mother and 40 yo brother have HTN. BMI is 23. HR 82/min, BP 144/92. No other abnormalities. Next step? This one is tricky. Since the guy appears "normal" in terms of appearence with the exception of his BP, I would probably have him come back again just to manage it and more readings.

a. routine exam in 1 yr
b. decreased dietary intake of sodium (INCORRECT)
c. 24 hr urine collection for measurement of cathecholamines
d. second measurement of blood pressure in two weeks
e. exercise stress test

47 yo M with 2 yr hx of increasing left knee pain. Initially pain was only felt when playing basketball or running, but now he has pain all the time. During the last 6 months his pain has been waking him up at night. Pain is worse with walking or prolonged standing. He used to take part in sports 2-3x weely but now has stopped participating due to pain. When he was 16 he fractured his left proximal tibia while playing football for at which point he underwent open reduction and internal fixation. Says he occasionally has swelling of knee but has not had any locking or catching. There is a varus deformity of left knee on exam. Anterior and posterior drawer tests are are negative. Explanation for these findings? Since none of the answer choices except one made sense to me I would say C is the most likely correct choice. I also just happen to know some runners and football player with injuries that occured in childhood who complain about pain and answer choice C fits with their presentation as well

a. nonunion of his prior fracture
b. patella tendinitis (INCORRECT)
c. post traumatic arthritis
d. tear of anterior cruciate ligament
e. tear of medial meniscus

37 yo F G2P2 with 3 month hx of SOB with exertion with hx of rheumatic fever during childhood. Prenatal exams 5 and 10 yrs ago revealed a murmur, but otherwise both pregnancies were uncomplicated. She takes no meds, and does not smoke or drink alcohol. BMI is 19. BP 105/70, HR 70/min and irregular, RR 18/min, 98.2 F. There is malar flushing and JVD on exam and lungs are clear to auscultation. A grade 2/6 late diastolic murmur is heard at apex. There is a loud S1. Cause of patient's dyspnea?

a. Decreased left atrial pressure
b. Decreases left ventricular pressure
c. Decreased pulm venous pressure
d. Increased left ventricular pressure (INCORRECT)
e. Increased pulm venous pressure

4 yo boy with2 week hx of increasing left ear pain. Temp 101.3 F. Left ear has erythematous, bulging tympanic membrane and an edematous auditory canal with a small amount of mucopurulent discharge. Ear appears to be displaced laterally with moderate tenderness behind the ear. No abnormalities in right ear. A 1 cm nontender, freely mobile mass is palpated over left anterior neck. No abnormalities in pharynx. Next step in managment?

a. monospot test
b. tympanography
c. CT scan of temporal bone
d. hydrocortisone/polymyxin/neomycin ear drops
e. oral amoxicillin therapy (INCORRECT)

47 yo M in ER severe neck pain 2 hrs after lifting a heavy tool at his contruction job. Pain is worse when he turns his neck or coughs and now radiates over his right shoulder an arm. He has a 3 yr hx of intermittent neck pain that is better with ibuprofen. Currently he is in severe distress. Finds it difficult to turn his head; when asked to do so he turns it slowly due to pain. Muscle strength is 4/5 in elbow flexor and wrist extensor muscles on the right. Right biceps and brachoradialis muscles have decreased DTRs. Cause of these findings?

a. central cord syndrome
b. compression of lower brachial plexus
c. compression of musculocutaneous nerve
d. compression upper brachial plexus (INCORRECT)
e. herniated disk at C5-6
 
Last edited:
Thank you soooo much @cruCiATE and @bluemagik5 !!! I really appreciate your input.

I have 2 questions, one with a pic and one from biostats. I've attached the pic below.

23 yo uncomfortable appearing F, 3 mo hx of severe headaches. Due to the pain she has been unable to go to work for several days. She reports a 10 yr hx of intermittent headaches that respond to naproxen and major depression treated with flouxetine. BMI 20. Bilateral papilledema is seen on fundoscope. Pupils are 5 mm and reactive to light. MRI of the brain is shown (pic 1). Cause of findings? (What is this picture showing? My first thought was glioblastoma since the density is bilateral but idk.)

a. idiopathic intracranial HTN
b. impaired resorption of CSF (INCORRECT)
c. infection of CSF
d. obstruction of ventricular system
e. overproduction of CSF

Double blind study on the effectiveness of hormone replacement therapy (HRT) on incidence of cancer in post menopausal women. The study population consists of 16,408 postmenopausal women who have not undergone hysterectomies and are randomly selected to receive either HRT or a placebo. After 6 yrs the following results are obtained:

Cancer # of cases in both groups Hazard ratio for 95% confidence interval
HRT group vs. Placebo group

Ovarian 62 1.58 1.2 to 3.24
Endometrial 58 0.81 0.48 to 1.36
Cervical 13 0.8 0.5 to 1.2

a. HRT decreases incidence of ovarian cancer (INCORRECT)
b. HRT increases incidence of ovarian cancer
c. HRT has no effect on the incidence of ovarian cancer
d. HRT has a protective effect against ovarian cancer
e. No conclusion can be made based on the given data

Can you explain what hazard ratio means and the thought process of answering this? I thought 1.58 meant an increased risk because it is more than one and the confidence interval of 1.2 to 3.24 was significant because it didn't include 1.
 

Attachments

  • pic 1.jpg
    pic 1.jpg
    57.7 KB · Views: 69
23 yo uncomfortable appearing F, 3 mo hx of severe headaches. Due to the pain she has been unable to go to work for several days. She reports a 10 yr hx of intermittent headaches that respond to naproxen and major depression treated with flouxetine. BMI 20. Bilateral papilledema is seen on fundoscope. Pupils are 5 mm and reactive to light. MRI of the brain is shown (pic 1). Cause of findings? (What is this picture showing? My first thought was glioblastoma since the density is bilateral but idk.)

a. idiopathic intracranial HTN (pseudomotor cerebri, Obese young woman)
b. impaired resorption of CSF (INCORRECT) (this would be the case for Normal Pressure Hydrocephalus, Old man with Dementia, Ataxia and Urinary Incontinence)
c. infection of CSF
d. obstruction of ventricular system (tumor is obstructing the ventricular system and that is causing increase intracranial pressure)
e. overproduction of CSF

Double blind study on the effectiveness of hormone replacement therapy (HRT) on incidence of cancer in post menopausal women. The study population consists of 16,408 postmenopausal women who have not undergone hysterectomies and are randomly selected to receive either HRT or a placebo. After 6 yrs the following results are obtained:

Cancer # of cases in both groups Hazard ratio for 95% confidence interval
HRT group vs. Placebo group

Ovarian 62 1.58 1.2 to 3.24
Endometrial 58 0.81 0.48 to 1.36
Cervical 13 0.8 0.5 to 1.2

a. HRT decreases incidence of ovarian cancer (INCORRECT)
b. HRT increases incidence of ovarian cancer (Hazard ratio for ovarian cancer is 1.58 while on HRT, since is more than 1 is saying that there is a correlation between Ovarian Cancer and Pt. on HRT vs .58 of the Placebo so almost double)
c. HRT has no effect on the incidence of ovarian cancer
d. HRT has a protective effect against ovarian cancer
e. No conclusion can be made based on the given data

Hope it helps
 
What I'm confused about is when they say "Hazard ratio for HRT group vs. Placebo group is 1.58" are they saying that since HRT is written first the ratio is

HRT group: Placebo group and thus HRT is more hazardous?

For example if it was written "Hazard ratio for Placebo group vs. HRT group is 1.58" then since Placebo is written first the ratio is

Placebo group: HRT group is 1.58..... in which case placebo would be more hazardous than HRT?

Am I right? @cruCiATE
 
77 yo F, severe SOB after colectomy 3 days ago. Current meds: subcutaneous heparin & patient-controlled morphine. HR 140/min, resp 28/min, temp 98.6 F, BP 110/60, O2 sat: 89%. Breath sounds are reduced on right. Surgical incision as no discharge or erythema. ABGs: pH 7.38, PCO2 23, PO2 55. Atelectasis at both lung bases on CXR.
Nonspecific ST-T wave changes on EKG. Besides O2 theraphy, next step in mangment?

a. Spiral CT scan of chest (check for PE)
b. oral warfarin therapy
c. IV recombinant tPA
d. Pulm angiography (INCORRECT)
e. Placement of IVC filter

37 yo M who returned form a trip to Mexico 1 mo ago, with fever & bilateral flank pain. Takes no meds & has fever (102.2 F). Spleen is palpable 2 cm under coastal margin with deep inspiration. Hb 9, reticulocytes 5%, LDH 300. Next step in Diagnosis?

a. Aspiration of lesion
b. examination of duodenal aspirate for ova & parasites
c. examination of stool for ova & parasites (INCORRECT)
d. serum antibody titer for Entameba histolytica (Mexico, Hepatomegaly, posible amebic liver abscess, Histolycia = increase LDH)
e. serum antibody titer for Plasmodium species
f. serum antibody titer for strongyloides stercoralis
g. serum antibody titer for visceral larva migrans
h. skin test for trichinella spiralis
i. thick and thin blood smears for malaria

77 yo F 24 hr hx of watery diarrhea & nausea is admitted. So far during her stay she has vomited 4x and has not been able to drink or eat. Patient has hx of HTN, DM2, hyperlipidemia, chronic renal insufficiency. She is taking atorvastatin, lisinopril, HCTZ, & aspirin. She has dry mucous membranes. Abdomen is non-tender & soft. Vitals: BP 100/60, HR 110/min, temp 99.3 F, RR 22/min. Labs: Na+ 132, K+ 8.2, Cl- 99, HCO3 18, BUN 95, Creatinine 6.4, Glucose 199. Peaked T waves and QRS of 0.16 on EKG. Next step in managment?

a. administer bicarbonate
b. administer calcium gluconate (Hyperkalemia with EKG Changes, Calcium to stabilize Cardiac membrane, prevent life threatening arrhythmia)
c. administer insulin and 50% dextrose in water
d. administer sodium polystyrene sulfonate (Kayexalate)
e. Schedule dialysis (INCORRECT - I thought she had hyperkalemia and renal failure so that was an indication for dialysis???)

,
23 yo F with 4 days hx of abd discomfort, loss of appetite, nausea. There is diffuse abd tenderness to deep palpation. RR 28/min. Labs: Na+ 132, Cl- 102, K+ 6, HCO3 10, Glucose 450. Cause?

a. addison's disease (INCORRECT)
b. diabetes insipidus
c. diabetic ketoacidosis (increase anion game metabolic acidosis, increase K+ due to decrease insulin, nausea classic for DKA)
d. hemolysis
e. laxative abuse
f. lower GI tract fistula
g. renal tubular acidosis
h. SLE
i. tumor lysis synd
j. vomiting

25 yo F, 4 hr hx of SOB, marked malaise, weakness, & fatigue. 24 hrs ago she had a strenous physical workout follwoed by muscle soreness. She took ibuprofen for her pain 1 hr prior to onset of her symptoms. Has hx of migraines for which she's been metoprolol 2x daily for pro[hylaxis over the past month. Exam: mild periorbital edema & facial flushing. There are multiple wheezes through out lung fields. BP 100/70. Substance to prevent recurrence of similar episode?

a. acetaminophen (INCORRECT)
b. aspirin (Ibuprofen probably caused this reaction, so avoid any NSAIDs, you don't want to shift the arachidonic pathway towards the LKTs)
c. meperidine
d. prednisone
e. propoxyphene

87 yo nursing home resident, with fever for 1 day. She has urinary incontinence for which a catheter was placed 2 weeks ago. Hx of Alzheimers dementia and cannot verbally communicate. Has moist and pink mucous membranes. Vitals: HR 86/min, BP 120/74, resp 14/min, temp 100 F. Urinanalysis: cloudy brown, pH 8.8, blood 2+,
Glucose -ve, protein 2+, RBC: too numerous to count, WBC 20-25, Nitrites 3+, Leukocyte esterase 3+, bacteria: many. Gram -ve bacilli on gram stain. Measure most likely to prevent patient's current condition?

a. use of incontinence briefs instead of catheter (avoid catheterization whenever possible to reduce the possibility of compilations)
b. changing catheter daily (INCORRECT)
c. cleaning urethral orifice with povidone iodide daily
d. flushing the catheter with antibiotic solution daily
e. oral antibiotic prophylaxis
f. oral oxybutynin therapy

57 yo F, hx of progressive cough productive of 2-3 tablespoons of yellow sputum daily for 2 yrs. It lasts all day but it worse in the morning. She is unable to walk more than one block before becoming short of breath. No hx of fever, hemoptysis, weight loss, or night sweats. Smoking hx of 2 packs daily for 40 yrs. BMI 32. BP 150/90, HR 9/min, RR 20/min. Has an O2 saturation of 90%. Breath sounds are decreased in all lung fields with rhonchi heard at bases and a prolonged expiratory phase. She has sinus tachycardia and P pulmonale on EKG. PFTs show FEV1 greater than 80% of predicted, & FEV1/FVC ratio of 70% of predicted and a diffusion lung capacity of carbon monoxide greater than 60 % of predicted. Diagnosis?

a. chronic bronchitis (2 years of cough, had the same feeling with DLCO but I am assuming the just tried to say is 60% of expected, ie. less)
b. cystic fibrosis
c. emphysema (INCORRECT - I thought only emphysema had an increased CO diffusion capacity?)
d kartegener's syndrome
e. post nasal drip syndrome ie.

42 yo M, MVA, undergoes splenectomy and one hour later has severe SOB. Other injuries are left rib and pelvic fractures. Vitals: BP 80/60, HR 133/min, RR 35/min, temp 97.4 F. He has slight distention & absent bowel sounds on abd exam. Next step in management.

a. Xray of chest
b. transfusion type specific uncrossedmatched blood
c. IV heparin therapy
d. fiberoptic bronchoscopy
e. endotracheal intubation (INCORRECT)
f. placement of vena cava filter
g. needle thoracostomy (pt is unstable, Pneumothorax probably after rib fracture, Hypotension, Tachycardia, Tachypnea, decompress with need then chest tube)


72 yo F 3 mo hx of bilateral knee pain with weight bearing. OTC naproxen at maximum dose over past 6 weeks has partially releived the pain. Has Hx of HTN, CAD, CHF, & osteoporosis. Also has hx of stable exertional angina & moderate SOB with exertion. She is taking atenolol, potassium supplements, furosemide. BP 115/70, HR 62/min. She has JVD and lungs are clear to auscultation. CVS exam shows no murmurs or gallops but she has pedal edema. She has crepitus of both knees, and bony prominences at proximal tibial bone on both sides. Labs show potassium of 5.1, BUN of 47 (baseline 20), creatinine of 2.2 (baseline of 1). Which medication is responsible for abnormal lab findings?

a. atenolol
b. furosemide
c. naproxen (naproxen for 6 weeks high does, increase BUN, Cr, NSAID dame the kidney tubules)
d. nitrates
e. potassium supplements (INCORRECT)

67 yo alcoholic male comes for routine exam. Has a 15 yr hx of poorly controlled HTN for which he takes HCTZ but he is not compliant. His blood pressure at the current visit is 170/102 and was at this level for the last 3 office visits. Funduscope findings: AV nicking & tortuosity of the arteries. Patient is at greatest risk for?

a. aortic aneurysm (INCORRECT) (not a smoker)
b. glaucoma
c. myocardial infarction (pt. has risk factors for MI)
d. pulmonary HTN
e. subarachnoid hemorrage

67 yo African American F comes for routine exam. She reached menopause at age 44. She has hypothyroidism and is taking thyroid replacement therapy. Has smoking hx of 1 pack/daily for 45 yrs. BMI 21. Which factor decreases patient's risk for osteoporosis?

a. African American race (increase bone density)
b .cigarette smoking
c. early menopause
d. thin body habitus
e. thyroid replacement therapy (INCORRECT)

37 yo F with known ITP has sudden onset of fever 7 days after undergoing splenectomy. She has had 2 days of mild shoulder pain & moderate abd pain. Hb has been between 9.5 and 10 since the splenectomy. Currently she is on hydrocodone and docusate. She has a fever of 102.4 F. Abd Exam; mild distention & diffuse tenderness without rebound, rigidity, or guarding, and absent bowel sounds. Labs: Hb 9.8, Leukocyte ct. 21,300, segmented neutrophils 50%, bands 17%, eosinophils 3%, lymphocytes 25%, monocytes 5%, platelet ct 105,000, amylase 124. Xray shows left pleurual effusion. Cause of findings?

a. aspiration
b. gastric perforation (INCORRECT) (no air under the diapharm)
c. pancreatitis
d. pneumonia
e. subphrenic abscess (Post-splenectomy subphrenic abscess, phrenic nerve impingement cause refered shoulder pain, abscess fits the Fever, Increase Leukocyte count)

82 yo M, lives in nursing home: SOB at rest, increasing malaise, generalized weakness, & depressed mood over past 6 weeks. He bruises easily and has bleeding & infla)mmation of the gums. He reports a minor skin abrasion 1 month ago that has not healed completely, and he appears chronically ill. Exam: numerous petechiae, eccymosis, & hypertrophic lichenified patches over upper extremities. Mild pedal edema is present. He is oriented to TPP and has no short or long term memory loss. Labs including CBC, urinanalyis, measurement of serum electrolyte concentrations, are all with in normal limits. Diagnosis?

a. Acquired factor VIII deficiency (INCORRECT)
b. hypersensitivity vasculitis
c. polymyalgia rheumatica
d. psoriasis
e. thrombocytopenic purpura
f. vit B12 deficiency
g. vit C deficiency (Scurvy, malnutrition, easy bruising, bleeding gums)

Asymptomatic 72 yo F for routine exam, takes no meds. DEXA findings at age 65 were normal. At her last visit 1 yr ago pap smear, mammogrpahy, sigmoidoscopy, were all normal. Her serum cholesterol was 190 and glucose was 80. She remains sexually active and does not smoke. She drinks 1-2 glasses of wine 3x weekly. BMI 23. BP 140/85. Physical and pelvic exams show no abnormalities. Next step in managment?

a. measurment of fasting glucose concentration
b. serum lipid studies (INCORRECT)
c. ECG
d. Influenza virus vaccination
e. sigmoidoscopy

72 yo lethargic appearing dementia patient who lives in a nursing home brought to ER 2 days after onset of fever, confusion, and generalized weakness. Symptoms began following irrigation of his urinary catheter, which wasn't draining well. He has hx of COPD and osteoarthritis. Vitals: Temp 97.2 F, HR 110, BP 90/50. Has supple neck on exam. Herbenden nodes are seen on the hands. Breath sounds are decreased throughout lungs with no dullness. Cranial nerves are intact and DTRs are normal. Labs: Hct 50%, leukocyte ct 15,400, segmented neutrophils 74%, bands 4%, lymphocytes 22%, Na+ 134, K+ 4.2, Cl- 86, HCO3 24, BUN 12, creatinine 1.1. Sinus tachycardiea and some flattening of ST-T segments. Most likely explanation for hypotension?

a. decreased cardiac contractility
b. decreased systemic vascular resistance (Septic shock secondary to UTI)
c. deficiency of mineralcorticoids
d. impaired vagal reflexes
e. impaired venous return (INCORRECT)

Follow up of a 6 mo boy who's had chronic constipation since age 1 week. Current treatment with rectal stimulation, glycerin suppositories, & 4 oz of prune juice produces one firm string like stool every 4 days. After being breast fed until 2 weeks of age he was switched to cow's milk-based formula. At the moment his diet also consists of cereals, fruits, pureed vegetables. Abd exam: distention with no tenderness. Rectal exam shows no palpable stool in ampulla. Next step in management?

a. switch to lactose-free formula
b. test of stool for botulism toxin (INCORRECT)
c. stool culture
d. measurement of serum TSH concentration
e. rectal manometry (Failure to pass Meconium in a neonate is Hirshprung dz until proven otherwise, do Manometry)
f. upper GI series

52 yo F, routine exam. She was started on daily hormone replacment therapy (estradiol and medroxyprogesterone) one month ago to treat severe vasomotor symptoms. But she has been taking the medroxyprogesterone intermittently due to severe depression and mood changes while on it. For the first 6 mo of the regimen menses had stopped. However, since she changed her regimen she has vaginal spotting 2 -3x/ monthly. Next step in managment?

a. discontinue medroxyprogesterone therapy only
b. increase dosage of estradiol
c. add an antidepressant to the medication regimen (INCORRECT)
d. endometrial abalation
e. endometrial biopsy (vaginal spoting after HRT (1+mo) can be Endometrial Hyperplasia you need to Biopsy to rule out pre-malignany or malignancy)


3 yo girl, 1 day hx of fever and ear pain, has had clear nasal discharge and cough for 3 days. Over last 12 months she has had a hx of several ear infections and one episode of strep pharangitis. Her family has two cats and the father smokes in the home. She frequently swims in the family's swimming pool. BP 80/50, HR 110/min, RR 22/min, temp 101.3 F. She has clear nasal discharage, erythemea and bulging of right tympanic membrane and erythema of throat without exudate. Lungs are clear toa, auscultation. Most appropriate recommendation to prevent recurrence of this patient's condition?

a. avoidance of passive smoke exposure (research this one, not completely sure)
b. removal of cats from the house
c. use of earplugs while swimming (INCORRECT)
d. daily otic antibiotics
e. prophylactic oral decongestants
f. tonsilectomy

67 yo F, 2 mo hx of intermittent burning sensation in her epigastrium. It presents after she walks one block or climbs one flight of stairs and is sometimes associated with lightheadedness, and profuse sweating. Epigastric discomfort is better with rest and is not relieved by antacids. Her meds include daily multivitamin and calcium supplements. Smoker for 40 yrs with 1 pack/daily. She has one drink of alcohol 1-2x/ weekly. BP 150/90, HR 98/min, resp 15/min, temp 98.6F. Results of labs, including cardiac enzymes, are within normal range. No abnormalities are noted on EKG. Most appropriate next step in diagnosis?

a. ambulatory EKG monitoring (INCORRECT)
b. abdominal U/S
c. Cardiac stress schintography (riks factors for CAD, epigastric=vague way of saying substernal, stable angina, do exercise to reproduce the ischemia check EKG for any ST segment changes)
d. 24 hr monitoring of esophageal pH
e. esophagogastroduodenoscopy

62 yo M brought 4 hrs after the sudden onset of vomiting & severe abd pain. Appears diaphoretic and in acute distress. He has had mild,intermittent epigastric pain over past year that is relieved by antacids. No hx of serious illness. He is a smoker : 1 pack/daily for 35 yrs and has 1-2 drinks/ daily. BP 115/75, HR 100/min, RR 18/min, temp 101.1 F. Abd is rigid and tympanic to percussion over the liver, with severe diffuse tenderness. No bowel sounds are present. Next step in diagnosis?

a. X-rays of chest and abdomen (severe pain, pain over past year relieve by anti acids=gastritis, this on is probably a perforation)
b. barium enema
c. water soluble contrast swallow
d. CT scan of abd (INCORRECT)
e. colonoscopy
f. flexible esophagogastroduodenoscopy

32 yo F, 2 day hx of fever & an area of swelling & redness on her right forearm. She noticed red streaks extending from her forearm to her elbow 6 hrs ago. No travel or trauma hx. Temp 100 F. Epitrochlear lymph nodes are enlarged. At the distal radial aspect of right upper extremity, an 8 x 13 warm, erythematous area of edema that is tender to palpation is seen. There is a warm, tender red streak extending from this area to the elbow. Organism?

a. H. influenzae
b. B-hemolytic strep (cellulitis)
c. psuedomonas aeruginosa
d. salmonella choleraesuis
e. staph epidermidis (INCORRECT) (no trauma Hx or prosthetic divice)

Follow up of 47 yo M after being diagnosed with stage 1 HTN 5 months ago. HTN has not been controlled despite 3 mo trial of diet modification and exercise. HCTZ therapy was begun. He has felt fatigued over the past month. No abnormalities on noted on exam. Cause of patient's symptoms?

a. hyperglycemia
b. hyperlipidemia
c. hypocalcemia
d. hypokalemia (weakness)
e. hyponatremia (INCORRECT) (this will cause CNS problems)

52 yo F, G2P2, 2 yr hx of increasingly frequent loss of urine. She feels an overwhelming urge to void and completely saturates an absorbant pad before reaching the bathroom. The most frequent occurrence of loss of urine is when she steps out of the car. She wakes up 3-4x a night to urinate. No loss of urine with coughing or sneezing. She is being treated with a diuretic for HTN. Vitals are normal. Exam of abdomen, external genitalia,vagina, and cervix are all normal. Uterus and adexna are normal on palpation. No loss of urine with valsalva or coughing. Post void volume is 50 mL. Urinanalysis is normal. Diagnosis?

a. overactive bladder with incontinence (urge incontinence)
b. overflow incontinence (INCORRECT)
c. stress incontinence
d. UTI
e. vesicovaginal fistula

72 yo M in the ER; has vomited 3x within the last 12 hrs and has also felt dizzy. He has been taking ibuprofen daily for arthritis since 5 yrs. He increased the dose of ibuprofen in the past 4 weeks due to increasing joint pain but he does not recall how many pills he took. Due to his joint pain he is unable to take his daily walks. He does not have chest pain. He had an uncomplicated MI 10 yrs ago. He looks pale. BP 90/60, HR 110, RR 24/min, temp 99.5 F. Dried blood is seen near the mouth. Lungs are clear to auscultation and heart sounds are normal. Stool appears black and is positive for occult blood. Hematocrit is 22% and platelet ct 215,000. Hemorrhagic gastritis with no active site of bleeding is seen on upper GI endoscopy. Patient becomes short of breath 3o minutes after fluid resuscitation with crystalloid solution and transfusion of 4 units of crossmatched packed RBCs. Diffuse rhonchi and crackles are heard bilaterally. Cause of dyspnea?

a. ABO incompatibility
b. Acute resp distress syndrome (INCORRECT)
c. Aspiration of gastric contents
d. Fluid overload (pulmonary edema due to excessive fluid resuscitation)
e. pulmonary embolism

43 yo M, 3 mo hx of fatigue, generalized weakness, & depressed mood; has also experienced cold intolerance, muscle aches, & constipation. He is a strict vegetarian. His wife has hypothyroidism treated with levothyroxine. BP 120/95, HR 62/min, temp 98.6 F. He has dry skin. Thyroid is not enlarged and testes are small. His Achilles tendon reflex has a delayed relaxation phase. His TSH is 0.01 microunits/mL, T4 is 2.3 microunits/dL. Mechanism of findings?

a. autoantibodies against thyroglobulin
b. deficient pituitary production of TSH (hypothyroidism, Pituitary defect)
c. dietary deficiency of iodine
d. increased production of thyroid binding globulin
e. surreptitious levothyroxine use (INCORRECT)

Asymptomatic 37 yo African American M, preemployment exam, with no hx of serious illness and is not on any meds. He is not a smoker and exercises regularly. Both his mother and 40 yo brother have HTN. BMI is 23. HR 82/min, BP 144/92. No other abnormalities. Next step?

a. routine exam in 1 yr
b. decreased dietary intake of sodium (INCORRECT)
c. 24 hr urine collection for measurement of cathecholamines
d. second measurement of blood pressure in two weeks (Two separate measurements are needed to Dx HTN)
e. exercise stress test

47 yo M with 2 yr hx of increasing left knee pain. Initially pain was only felt when playing basketball or running, but now he has pain all the time. During the last 6 months his pain has been waking him up at night. Pain is worse with walking or prolonged standing. He used to take part in sports 2-3x weely but now has stopped participating due to pain. When he was 16 he fractured his left proximal tibia while playing football for at which point he underwent open reduction and internal fixation. Says he occasionally has swelling of knee but has not had any locking or catching. There is a varus deformity of left knee on exam. Anterior and posterior drawer tests are are negative. Explanation for these findings?

a. nonunion of his prior fracture
b. patella tendinitis (INCORRECT)
c. post traumatic arthritis (fracture when he was 16yrs old, probably developed scar tissue and that accelerated arthritis)
d. tear of anterior cruciate ligament
e. tear of medial meniscus

37 yo F G2P2 with 3 month hx of SOB with exertion with hx of rheumatic fever during childhood. Prenatal exams 5 and 10 yrs ago revealed a murmur, but otherwise both pregnancies were uncomplicated. She takes no meds, and does not smoke or drink alcohol. BMI is 19. BP 105/70, HR 70/min and irregular, RR 18/min, 98.2 F. There is malar flushing and JVD on exam and lungs are clear to auscultation. A grade 2/6 late diastolic murmur is heard at apex. There is a loud S1. Cause of patient's dyspnea?

a. Decreased left atrial pressure
b. Decreases left ventricular pressure
c. Decreased pulm venous pressure
d. Increased left ventricular pressure (INCORRECT)
e. Increased pull venous pressure (Mitral stenosis is very common in pregnancy due to increase volume, she has the murmur in apex, loud S1, this cause increase Left atrial pressure, which causes back up of blood though the pulmonary Veins)

4 yo boy with2 week hx of increasing left ear pain. Temp 101.3 F. Left ear has erythematous, bulging tympanic membrane and an edematous auditory canal with a small amount of mucopurulent discharge. Ear appears to be displaced laterally with moderate tenderness behind the ear. No abnormalities in right ear. A 1 cm nontender, freely mobile mass is palpated over left anterior neck. No abnormalities in pharynx. Next step in managment?

a. monospot test
b. tympanography
c. CT scan of temporal bone (ear displace laterally, post auricular tenderness= Mastoiditis,which is is a complication of AOM, check temporal bone with CT)
d. hydrocortisone/polymyxin/neomycin ear drops
e. oral amoxicillin therapy (INCORRECT)

47 yo M in ER severe neck pain 2 hrs after lifting a heavy tool at his contruction job. Pain is worse when he turns his neck or coughs and now radiates over his right shoulder an arm. He has a 3 yr hx of intermittent neck pain that is better with ibuprofen. Currently he is in severe distress. Finds it difficult to turn his head; when asked to do so he turns it slowly due to pain. Muscle strength is 4/5 in elbow flexor and wrist extensor muscles on the right. Right biceps and brachoradialis muscles have decreased DTRs. Cause of these findings?

a. central cord syndrome
b. compression of lower brachial plexus
c. compression of musculocutaneous nerve
d. compression upper brachial plexus (INCORRECT)
e. herniated disk at C5-6

hope it helps
I concur.
 
For this question the patient actually went to KENYA and not Mexico. I wrote it wrong. So then is the answer "serum antibody titer for plasmodium species" or "thick and thin blood smears for malaria"?

37 yo M who returned form a trip to Mexico 1 mo ago, with fever & bilateral flank pain. Takes no meds & has fever (102.2 F). Spleen is palpable 2 cm under coastal margin with deep inspiration. Hb 9, reticulocytes 5%, LDH 300. Next step in Diagnosis?

a. Aspiration of lesion
b. examination of duodenal aspirate for ova & parasites
c. examination of stool for ova & parasites (INCORRECT)
d. serum antibody titer for Entameba histolytica (Mexico, Hepatomegaly, posible amebic liver abscess, Histolycia = increase LDH)
e. serum antibody titer for Plasmodium species
f. serum antibody titer for strongyloides stercoralis
g. serum antibody titer for visceral larva migrans
h. skin test for trichinella spiralis
i. thick and thin blood smears for malaria
 
For this question the patient actually went to KENYA and not Mexico. I wrote it wrong. So then is the answer "serum antibody titer for plasmodium species" or "thick and thin blood smears for malaria"?

37 yo M who returned form a trip to Mexico 1 mo ago, with fever & bilateral flank pain. Takes no meds & has fever (102.2 F). Spleen is palpable 2 cm under coastal margin with deep inspiration. Hb 9, reticulocytes 5%, LDH 300. Next step in Diagnosis?

a. Aspiration of lesion
b. examination of duodenal aspirate for ova & parasites
c. examination of stool for ova & parasites (INCORRECT)
d. serum antibody titer for Entameba histolyticae.
e. serum antibody titer for Plasmodium species
f. serum antibody titer for strongyloides stercoralis
g. serum antibody titer for visceral larva migrans
h. skin test for trichinella spiralis
i. thick and thin blood smears for malaria (Malaria = Plasmodium + Africa + Fever + Headache + Anemia + Splenomegaly + hemolysis) this one I checked on uptodate.
 
Can someone please explain this question (with reasoning for the wrong answer choices too) ? It's very confusing.

47 yo M in ER severe neck pain 2 hrs after lifting a heavy tool at his contruction job. Pain is worse when he turns his neck or coughs and now radiates over his right shoulder an arm. He has a 3 yr hx of intermittent neck pain that is better with ibuprofen. Currently he is in severe distress. Finds it difficult to turn his head; when asked to do so he turns it slowly due to pain. Muscle strength is 4/5 in elbow flexor and wrist extensor muscles on the right. Right biceps and brachoradialis muscles have decreased DTRs. Cause of these findings?

a. central cord syndrome
b. compression of lower brachial plexus
c. compression of musculocutaneous nerve
d. compression upper brachial plexus (INCORRECT)
e. herniated disk at C5-6
 
37 yo F G2P2 with 3 month hx of SOB with exertion with hx of rheumatic fever during childhood. Prenatal exams 5 and 10 yrs ago revealed a murmur, but otherwise both pregnancies were uncomplicated. She takes no meds, and does not smoke or drink alcohol. BMI is 19. BP 105/70, HR 70/min and irregular, RR 18/min, 98.2 F. There is malar flushing and JVD on exam and lungs are clear to auscultation. A grade 2/6 late diastolic murmur is heard at apex. There is a loud S1. Cause of patient's dyspnea?

a. Decreased left atrial pressure
b. Decreases left ventricular pressure
c. Decreased pulm venous pressure
d. Increased left ventricular pressure (INCORRECT)
e. Increased pull venous pressure (Mitral stenosis is very common in pregnancy due to increase volume, she has the murmur in apex, loud S1, this cause increase Left atrial pressure, which causes back up of blood though the pulmonary Veins)

Why would the S1 be loud in Mitral Stenosis, if anything it should be quieter. Since you have less blood in the ventricle, and then there will be less blood pushing against the Mitral valve to close it. I dont understand what is causing loud S1..


77 yo F, severe SOB after colectomy 3 days ago. Current meds: subcutaneous heparin & patient-controlled morphine. HR 140/min, resp 28/min, temp 98.6 F, BP 110/60, O2 sat: 89%. Breath sounds are reduced on right. Surgical incision as no discharge or erythema. ABGs: pH 7.38, PCO2 23, PO2 55. Atelectasis at both lung bases on CXR.
Nonspecific ST-T wave changes on EKG. Besides O2 theraphy, next step in mangment?

a. Spiral CT scan of chest (check for PE)
b. oral warfarin therapy
c. IV recombinant tPA
d. Pulm angiography (INCORRECT)
e. Placement of IVC filter

I think Pulmonary Angiography is correct, at least according to this algorithm.
http://www.aafp.org/afp/2001/0901/afp20010901p844a-uf1.gif
afp20010901p844a-uf1.gif
 
What are your thoughts on this one:

13 yr old boy has 3 month hx of left knee pain that exacerbates by exercise. He also sometimes has knee pain in his RIGHT knee. No trauma. No fever. 50th percentile for weight an height. ROM of both knees normal. What is the dx and next step:

Bone scan
MRI of left knee
Antibiotics
NSAIDs
Knee Immobilization
Joint aspiration
 
Follow up of a 6 mo boy who's had chronic constipation since age 1 week. Current treatment with rectal stimulation, glycerin suppositories, & 4 oz of prune juice produces one firm string like stool every 4 days. After being breast fed until 2 weeks of age he was switched to cow's milk-based formula. At the moment his diet also consists of cereals, fruits, pureed vegetables. Abd exam: distention with no tenderness. Rectal exam shows no palpable stool in ampulla. Next step in management?

a. switch to lactose-free formula
b. test of stool for botulism toxin (INCORRECT)
c. stool culture
d. measurement of serum TSH concentration
e. rectal manometry (Failure to pass Meconium in a neonate is Hirshprung dz until proven otherwise, do Manometry)
f. upper GI series



Isn't that rule for the first 48 hrs of birth? I didn't pick rectal manometry since the kid is 6 month old. So I picked d.hypothyroidism (incorrect)
77 yo F, severe SOB after colectomy 3 days ago. Current meds: subcutaneous heparin & patient-controlled morphine. HR 140/min, resp 28/min, temp 98.6 F, BP 110/60, O2 sat: 89%. Breath sounds are reduced on right. Surgical incision as no discharge or erythema. ABGs: pH 7.38, PCO2 23, PO2 55. Atelectasis at both lung bases on CXR.
Nonspecific ST-T wave changes on EKG. Besides O2 theraphy, next step in mangment?

a. Spiral CT scan of chest (check for PE)
b. oral warfarin therapy
c. IV recombinant tPA
d. Pulm angiography (INCORRECT)
e. Placement of IVC filter

37 yo M who returned form a trip to Mexico 1 mo ago, with fever & bilateral flank pain. Takes no meds & has fever (102.2 F). Spleen is palpable 2 cm under coastal margin with deep inspiration. Hb 9, reticulocytes 5%, LDH 300. Next step in Diagnosis?

a. Aspiration of lesion
b. examination of duodenal aspirate for ova & parasites
c. examination of stool for ova & parasites (INCORRECT)
d. serum antibody titer for Entameba histolytica (Mexico, Hepatomegaly, posible amebic liver abscess, Histolycia = increase LDH)
e. serum antibody titer for Plasmodium species
f. serum antibody titer for strongyloides stercoralis
g. serum antibody titer for visceral larva migrans
h. skin test for trichinella spiralis
i. thick and thin blood smears for malaria

77 yo F 24 hr hx of watery diarrhea & nausea is admitted. So far during her stay she has vomited 4x and has not been able to drink or eat. Patient has hx of HTN, DM2, hyperlipidemia, chronic renal insufficiency. She is taking atorvastatin, lisinopril, HCTZ, & aspirin. She has dry mucous membranes. Abdomen is non-tender & soft. Vitals: BP 100/60, HR 110/min, temp 99.3 F, RR 22/min. Labs: Na+ 132, K+ 8.2, Cl- 99, HCO3 18, BUN 95, Creatinine 6.4, Glucose 199. Peaked T waves and QRS of 0.16 on EKG. Next step in managment?

a. administer bicarbonate
b. administer calcium gluconate (Hyperkalemia with EKG Changes, Calcium to stabilize Cardiac membrane, prevent life threatening arrhythmia)
c. administer insulin and 50% dextrose in water
d. administer sodium polystyrene sulfonate (Kayexalate)
e. Schedule dialysis (INCORRECT - I thought she had hyperkalemia and renal failure so that was an indication for dialysis???)

,
23 yo F with 4 days hx of abd discomfort, loss of appetite, nausea. There is diffuse abd tenderness to deep palpation. RR 28/min. Labs: Na+ 132, Cl- 102, K+ 6, HCO3 10, Glucose 450. Cause?

a. addison's disease (INCORRECT)
b. diabetes insipidus
c. diabetic ketoacidosis (increase anion game metabolic acidosis, increase K+ due to decrease insulin, nausea classic for DKA)
d. hemolysis
e. laxative abuse
f. lower GI tract fistula
g. renal tubular acidosis
h. SLE
i. tumor lysis synd
j. vomiting

25 yo F, 4 hr hx of SOB, marked malaise, weakness, & fatigue. 24 hrs ago she had a strenous physical workout follwoed by muscle soreness. She took ibuprofen for her pain 1 hr prior to onset of her symptoms. Has hx of migraines for which she's been metoprolol 2x daily for pro[hylaxis over the past month. Exam: mild periorbital edema & facial flushing. There are multiple wheezes through out lung fields. BP 100/70. Substance to prevent recurrence of similar episode?

a. acetaminophen (INCORRECT)
b. aspirin (Ibuprofen probably caused this reaction, so avoid any NSAIDs, you don't want to shift the arachidonic pathway towards the LKTs)
c. meperidine
d. prednisone
e. propoxyphene

87 yo nursing home resident, with fever for 1 day. She has urinary incontinence for which a catheter was placed 2 weeks ago. Hx of Alzheimers dementia and cannot verbally communicate. Has moist and pink mucous membranes. Vitals: HR 86/min, BP 120/74, resp 14/min, temp 100 F. Urinanalysis: cloudy brown, pH 8.8, blood 2+,
Glucose -ve, protein 2+, RBC: too numerous to count, WBC 20-25, Nitrites 3+, Leukocyte esterase 3+, bacteria: many. Gram -ve bacilli on gram stain. Measure most likely to prevent patient's current condition?

a. use of incontinence briefs instead of catheter (avoid catheterization whenever possible to reduce the possibility of compilations)
b. changing catheter daily (INCORRECT)
c. cleaning urethral orifice with povidone iodide daily
d. flushing the catheter with antibiotic solution daily
e. oral antibiotic prophylaxis
f. oral oxybutynin therapy

57 yo F, hx of progressive cough productive of 2-3 tablespoons of yellow sputum daily for 2 yrs. It lasts all day but it worse in the morning. She is unable to walk more than one block before becoming short of breath. No hx of fever, hemoptysis, weight loss, or night sweats. Smoking hx of 2 packs daily for 40 yrs. BMI 32. BP 150/90, HR 9/min, RR 20/min. Has an O2 saturation of 90%. Breath sounds are decreased in all lung fields with rhonchi heard at bases and a prolonged expiratory phase. She has sinus tachycardia and P pulmonale on EKG. PFTs show FEV1 greater than 80% of predicted, & FEV1/FVC ratio of 70% of predicted and a diffusion lung capacity of carbon monoxide greater than 60 % of predicted. Diagnosis?

a. chronic bronchitis (2 years of cough, had the same feeling with DLCO but I am assuming the just tried to say is 60% of expected, ie. less)
b. cystic fibrosis
c. emphysema (INCORRECT - I thought only emphysema had an increased CO diffusion capacity?)
d kartegener's syndrome
e. post nasal drip syndrome ie.

42 yo M, MVA, undergoes splenectomy and one hour later has severe SOB. Other injuries are left rib and pelvic fractures. Vitals: BP 80/60, HR 133/min, RR 35/min, temp 97.4 F. He has slight distention & absent bowel sounds on abd exam. Next step in management.

a. Xray of chest
b. transfusion type specific uncrossedmatched blood
c. IV heparin therapy
d. fiberoptic bronchoscopy
e. endotracheal intubation (INCORRECT)
f. placement of vena cava filter
g. needle thoracostomy (pt is unstable, Pneumothorax probably after rib fracture, Hypotension, Tachycardia, Tachypnea, decompress with need then chest tube)


72 yo F 3 mo hx of bilateral knee pain with weight bearing. OTC naproxen at maximum dose over past 6 weeks has partially releived the pain. Has Hx of HTN, CAD, CHF, & osteoporosis. Also has hx of stable exertional angina & moderate SOB with exertion. She is taking atenolol, potassium supplements, furosemide. BP 115/70, HR 62/min. She has JVD and lungs are clear to auscultation. CVS exam shows no murmurs or gallops but she has pedal edema. She has crepitus of both knees, and bony prominences at proximal tibial bone on both sides. Labs show potassium of 5.1, BUN of 47 (baseline 20), creatinine of 2.2 (baseline of 1). Which medication is responsible for abnormal lab findings?

a. atenolol
b. furosemide
c. naproxen (naproxen for 6 weeks high does, increase BUN, Cr, NSAID dame the kidney tubules)
d. nitrates
e. potassium supplements (INCORRECT)

67 yo alcoholic male comes for routine exam. Has a 15 yr hx of poorly controlled HTN for which he takes HCTZ but he is not compliant. His blood pressure at the current visit is 170/102 and was at this level for the last 3 office visits. Funduscope findings: AV nicking & tortuosity of the arteries. Patient is at greatest risk for?

a. aortic aneurysm (INCORRECT) (not a smoker)
b. glaucoma
c. myocardial infarction (pt. has risk factors for MI)
d. pulmonary HTN
e. subarachnoid hemorrage

67 yo African American F comes for routine exam. She reached menopause at age 44. She has hypothyroidism and is taking thyroid replacement therapy. Has smoking hx of 1 pack/daily for 45 yrs. BMI 21. Which factor decreases patient's risk for osteoporosis?

a. African American race (increase bone density)
b .cigarette smoking
c. early menopause
d. thin body habitus
e. thyroid replacement therapy (INCORRECT)

37 yo F with known ITP has sudden onset of fever 7 days after undergoing splenectomy. She has had 2 days of mild shoulder pain & moderate abd pain. Hb has been between 9.5 and 10 since the splenectomy. Currently she is on hydrocodone and docusate. She has a fever of 102.4 F. Abd Exam; mild distention & diffuse tenderness without rebound, rigidity, or guarding, and absent bowel sounds. Labs: Hb 9.8, Leukocyte ct. 21,300, segmented neutrophils 50%, bands 17%, eosinophils 3%, lymphocytes 25%, monocytes 5%, platelet ct 105,000, amylase 124. Xray shows left pleurual effusion. Cause of findings?

a. aspiration
b. gastric perforation (INCORRECT) (no air under the diapharm)
c. pancreatitis
d. pneumonia
e. subphrenic abscess (Post-splenectomy subphrenic abscess, phrenic nerve impingement cause refered shoulder pain, abscess fits the Fever, Increase Leukocyte count)

82 yo M, lives in nursing home: SOB at rest, increasing malaise, generalized weakness, & depressed mood over past 6 weeks. He bruises easily and has bleeding & infla)mmation of the gums. He reports a minor skin abrasion 1 month ago that has not healed completely, and he appears chronically ill. Exam: numerous petechiae, eccymosis, & hypertrophic lichenified patches over upper extremities. Mild pedal edema is present. He is oriented to TPP and has no short or long term memory loss. Labs including CBC, urinanalyis, measurement of serum electrolyte concentrations, are all with in normal limits. Diagnosis?

a. Acquired factor VIII deficiency (INCORRECT)
b. hypersensitivity vasculitis
c. polymyalgia rheumatica
d. psoriasis
e. thrombocytopenic purpura
f. vit B12 deficiency
g. vit C deficiency (Scurvy, malnutrition, easy bruising, bleeding gums)

Asymptomatic 72 yo F for routine exam, takes no meds. DEXA findings at age 65 were normal. At her last visit 1 yr ago pap smear, mammogrpahy, sigmoidoscopy, were all normal. Her serum cholesterol was 190 and glucose was 80. She remains sexually active and does not smoke. She drinks 1-2 glasses of wine 3x weekly. BMI 23. BP 140/85. Physical and pelvic exams show no abnormalities. Next step in managment?

a. measurment of fasting glucose concentration
b. serum lipid studies (INCORRECT)
c. ECG
d. Influenza virus vaccination
e. sigmoidoscopy

72 yo lethargic appearing dementia patient who lives in a nursing home brought to ER 2 days after onset of fever, confusion, and generalized weakness. Symptoms began following irrigation of his urinary catheter, which wasn't draining well. He has hx of COPD and osteoarthritis. Vitals: Temp 97.2 F, HR 110, BP 90/50. Has supple neck on exam. Herbenden nodes are seen on the hands. Breath sounds are decreased throughout lungs with no dullness. Cranial nerves are intact and DTRs are normal. Labs: Hct 50%, leukocyte ct 15,400, segmented neutrophils 74%, bands 4%, lymphocytes 22%, Na+ 134, K+ 4.2, Cl- 86, HCO3 24, BUN 12, creatinine 1.1. Sinus tachycardiea and some flattening of ST-T segments. Most likely explanation for hypotension?

a. decreased cardiac contractility
b. decreased systemic vascular resistance (Septic shock secondary to UTI)
c. deficiency of mineralcorticoids
d. impaired vagal reflexes
e. impaired venous return (INCORRECT)

Follow up of a 6 mo boy who's had chronic constipation since age 1 week. Current treatment with rectal stimulation, glycerin suppositories, & 4 oz of prune juice produces one firm string like stool every 4 days. After being breast fed until 2 weeks of age he was switched to cow's milk-based formula. At the moment his diet also consists of cereals, fruits, pureed vegetables. Abd exam: distention with no tenderness. Rectal exam shows no palpable stool in ampulla. Next step in management?

a. switch to lactose-free formula
b. test of stool for botulism toxin (INCORRECT)
c. stool culture
d. measurement of serum TSH concentration
e. rectal manometry (Failure to pass Meconium in a neonate is Hirshprung dz until proven otherwise, do Manometry)
f. upper GI series

52 yo F, routine exam. She was started on daily hormone replacment therapy (estradiol and medroxyprogesterone) one month ago to treat severe vasomotor symptoms. But she has been taking the medroxyprogesterone intermittently due to severe depression and mood changes while on it. For the first 6 mo of the regimen menses had stopped. However, since she changed her regimen she has vaginal spotting 2 -3x/ monthly. Next step in managment?

a. discontinue medroxyprogesterone therapy only
b. increase dosage of estradiol
c. add an antidepressant to the medication regimen (INCORRECT)
d. endometrial abalation
e. endometrial biopsy (vaginal spoting after HRT (1+mo) can be Endometrial Hyperplasia you need to Biopsy to rule out pre-malignany or malignancy)


3 yo girl, 1 day hx of fever and ear pain, has had clear nasal discharge and cough for 3 days. Over last 12 months she has had a hx of several ear infections and one episode of strep pharangitis. Her family has two cats and the father smokes in the home. She frequently swims in the family's swimming pool. BP 80/50, HR 110/min, RR 22/min, temp 101.3 F. She has clear nasal discharage, erythemea and bulging of right tympanic membrane and erythema of throat without exudate. Lungs are clear toa, auscultation. Most appropriate recommendation to prevent recurrence of this patient's condition?

a. avoidance of passive smoke exposure (research this one, not completely sure)
b. removal of cats from the house
c. use of earplugs while swimming (INCORRECT)
d. daily otic antibiotics
e. prophylactic oral decongestants
f. tonsilectomy

67 yo F, 2 mo hx of intermittent burning sensation in her epigastrium. It presents after she walks one block or climbs one flight of stairs and is sometimes associated with lightheadedness, and profuse sweating. Epigastric discomfort is better with rest and is not relieved by antacids. Her meds include daily multivitamin and calcium supplements. Smoker for 40 yrs with 1 pack/daily. She has one drink of alcohol 1-2x/ weekly. BP 150/90, HR 98/min, resp 15/min, temp 98.6F. Results of labs, including cardiac enzymes, are within normal range. No abnormalities are noted on EKG. Most appropriate next step in diagnosis?

a. ambulatory EKG monitoring (INCORRECT)
b. abdominal U/S
c. Cardiac stress schintography (riks factors for CAD, epigastric=vague way of saying substernal, stable angina, do exercise to reproduce the ischemia check EKG for any ST segment changes)
d. 24 hr monitoring of esophageal pH
e. esophagogastroduodenoscopy

62 yo M brought 4 hrs after the sudden onset of vomiting & severe abd pain. Appears diaphoretic and in acute distress. He has had mild,intermittent epigastric pain over past year that is relieved by antacids. No hx of serious illness. He is a smoker : 1 pack/daily for 35 yrs and has 1-2 drinks/ daily. BP 115/75, HR 100/min, RR 18/min, temp 101.1 F. Abd is rigid and tympanic to percussion over the liver, with severe diffuse tenderness. No bowel sounds are present. Next step in diagnosis?

a. X-rays of chest and abdomen (severe pain, pain over past year relieve by anti acids=gastritis, this on is probably a perforation)
b. barium enema
c. water soluble contrast swallow
d. CT scan of abd (INCORRECT)
e. colonoscopy
f. flexible esophagogastroduodenoscopy

32 yo F, 2 day hx of fever & an area of swelling & redness on her right forearm. She noticed red streaks extending from her forearm to her elbow 6 hrs ago. No travel or trauma hx. Temp 100 F. Epitrochlear lymph nodes are enlarged. At the distal radial aspect of right upper extremity, an 8 x 13 warm, erythematous area of edema that is tender to palpation is seen. There is a warm, tender red streak extending from this area to the elbow. Organism?

a. H. influenzae
b. B-hemolytic strep (cellulitis)
c. psuedomonas aeruginosa
d. salmonella choleraesuis
e. staph epidermidis (INCORRECT) (no trauma Hx or prosthetic divice)

Follow up of 47 yo M after being diagnosed with stage 1 HTN 5 months ago. HTN has not been controlled despite 3 mo trial of diet modification and exercise. HCTZ therapy was begun. He has felt fatigued over the past month. No abnormalities on noted on exam. Cause of patient's symptoms?

a. hyperglycemia
b. hyperlipidemia
c. hypocalcemia
d. hypokalemia (weakness)
e. hyponatremia (INCORRECT) (this will cause CNS problems)

52 yo F, G2P2, 2 yr hx of increasingly frequent loss of urine. She feels an overwhelming urge to void and completely saturates an absorbant pad before reaching the bathroom. The most frequent occurrence of loss of urine is when she steps out of the car. She wakes up 3-4x a night to urinate. No loss of urine with coughing or sneezing. She is being treated with a diuretic for HTN. Vitals are normal. Exam of abdomen, external genitalia,vagina, and cervix are all normal. Uterus and adexna are normal on palpation. No loss of urine with valsalva or coughing. Post void volume is 50 mL. Urinanalysis is normal. Diagnosis?

a. overactive bladder with incontinence (urge incontinence)
b. overflow incontinence (INCORRECT)
c. stress incontinence
d. UTI
e. vesicovaginal fistula

72 yo M in the ER; has vomited 3x within the last 12 hrs and has also felt dizzy. He has been taking ibuprofen daily for arthritis since 5 yrs. He increased the dose of ibuprofen in the past 4 weeks due to increasing joint pain but he does not recall how many pills he took. Due to his joint pain he is unable to take his daily walks. He does not have chest pain. He had an uncomplicated MI 10 yrs ago. He looks pale. BP 90/60, HR 110, RR 24/min, temp 99.5 F. Dried blood is seen near the mouth. Lungs are clear to auscultation and heart sounds are normal. Stool appears black and is positive for occult blood. Hematocrit is 22% and platelet ct 215,000. Hemorrhagic gastritis with no active site of bleeding is seen on upper GI endoscopy. Patient becomes short of breath 3o minutes after fluid resuscitation with crystalloid solution and transfusion of 4 units of crossmatched packed RBCs. Diffuse rhonchi and crackles are heard bilaterally. Cause of dyspnea?

a. ABO incompatibility
b. Acute resp distress syndrome (INCORRECT)
c. Aspiration of gastric contents
d. Fluid overload (pulmonary edema due to excessive fluid resuscitation)
e. pulmonary embolism

43 yo M, 3 mo hx of fatigue, generalized weakness, & depressed mood; has also experienced cold intolerance, muscle aches, & constipation. He is a strict vegetarian. His wife has hypothyroidism treated with levothyroxine. BP 120/95, HR 62/min, temp 98.6 F. He has dry skin. Thyroid is not enlarged and testes are small. His Achilles tendon reflex has a delayed relaxation phase. His TSH is 0.01 microunits/mL, T4 is 2.3 microunits/dL. Mechanism of findings?

a. autoantibodies against thyroglobulin
b. deficient pituitary production of TSH (hypothyroidism, Pituitary defect)
c. dietary deficiency of iodine
d. increased production of thyroid binding globulin
e. surreptitious levothyroxine use (INCORRECT)

Asymptomatic 37 yo African American M, preemployment exam, with no hx of serious illness and is not on any meds. He is not a smoker and exercises regularly. Both his mother and 40 yo brother have HTN. BMI is 23. HR 82/min, BP 144/92. No other abnormalities. Next step?

a. routine exam in 1 yr
b. decreased dietary intake of sodium (INCORRECT)
c. 24 hr urine collection for measurement of cathecholamines
d. second measurement of blood pressure in two weeks (Two separate measurements are needed to Dx HTN)
e. exercise stress test

47 yo M with 2 yr hx of increasing left knee pain. Initially pain was only felt when playing basketball or running, but now he has pain all the time. During the last 6 months his pain has been waking him up at night. Pain is worse with walking or prolonged standing. He used to take part in sports 2-3x weely but now has stopped participating due to pain. When he was 16 he fractured his left proximal tibia while playing football for at which point he underwent open reduction and internal fixation. Says he occasionally has swelling of knee but has not had any locking or catching. There is a varus deformity of left knee on exam. Anterior and posterior drawer tests are are negative. Explanation for these findings?

a. nonunion of his prior fracture
b. patella tendinitis (INCORRECT)
c. post traumatic arthritis (fracture when he was 16yrs old, probably developed scar tissue and that accelerated arthritis)
d. tear of anterior cruciate ligament
e. tear of medial meniscus

37 yo F G2P2 with 3 month hx of SOB with exertion with hx of rheumatic fever during childhood. Prenatal exams 5 and 10 yrs ago revealed a murmur, but otherwise both pregnancies were uncomplicated. She takes no meds, and does not smoke or drink alcohol. BMI is 19. BP 105/70, HR 70/min and irregular, RR 18/min, 98.2 F. There is malar flushing and JVD on exam and lungs are clear to auscultation. A grade 2/6 late diastolic murmur is heard at apex. There is a loud S1. Cause of patient's dyspnea?

a. Decreased left atrial pressure
b. Decreases left ventricular pressure
c. Decreased pulm venous pressure
d. Increased left ventricular pressure (INCORRECT)
e. Increased pull venous pressure (Mitral stenosis is very common in pregnancy due to increase volume, she has the murmur in apex, loud S1, this cause increase Left atrial pressure, which causes back up of blood though the pulmonary Veins)

4 yo boy with2 week hx of increasing left ear pain. Temp 101.3 F. Left ear has erythematous, bulging tympanic membrane and an edematous auditory canal with a small amount of mucopurulent discharge. Ear appears to be displaced laterally with moderate tenderness behind the ear. No abnormalities in right ear. A 1 cm nontender, freely mobile mass is palpated over left anterior neck. No abnormalities in pharynx. Next step in managment?

a. monospot test
b. tympanography
c. CT scan of temporal bone (ear displace laterally, post auricular tenderness= Mastoiditis,which is is a complication of AOM, check temporal bone with CT)
d. hydrocortisone/polymyxin/neomycin ear drops
e. oral amoxicillin therapy (INCORRECT)

47 yo M in ER severe neck pain 2 hrs after lifting a heavy tool at his contruction job. Pain is worse when he turns his neck or coughs and now radiates over his right shoulder an arm. He has a 3 yr hx of intermittent neck pain that is better with ibuprofen. Currently he is in severe distress. Finds it difficult to turn his head; when asked to do so he turns it slowly due to pain. Muscle strength is 4/5 in elbow flexor and wrist extensor muscles on the right. Right biceps and brachoradialis muscles have decreased DTRs. Cause of these findings?

a. central cord syndrome
b. compression of lower brachial plexus
c. compression of musculocutaneous nerve
d. compression upper brachial plexus (INCORRECT)
e. herniated disk at C5-6

hope it helps
 
For the PE question : Acc to Well's criteria PE is likely in this patient , so shouldn't we directly go to the rx ? and since he is already on heparin i thought that was like failure of anticoagulant therapy and hence go ahead with IVC filter ? Why are we doing CT first ?
 
exactly my thoughts. Pt is already on heparin and wells criteria is clearly above 4 so why waste time re diagnosing?
 
Top